Elementary Education 5018

User Manual: 5018

Open the PDF directly: View PDF PDF.
Page Count: 62

Elementary Education:
Content Knowledge
5018
www.ets.org/praxis
The Praxis® Study Companion
The Praxis® Study Companion 2
Welcome to the Praxis® Study Companion
Welcome to The Praxis®Study Companion
Prepare to Show What You Know
You have been working to acquire the knowledge and skills you need for your teaching career. Now you are
ready to demonstrate your abilities by taking a Praxis® test.
Using The Praxis Series® Study Companion is a smart way to prepare for the test so you can do your best on test
day. This guide can help keep you on track and make the most efficient use of your study time.
The Study Companion contains practical information and helpful tools, including:
An overview of the Praxis tests
Specific information on the Praxis test you are taking
A template study plan
• Study topics
Practice questions and explanations of correct answers
Test-taking tips and strategies
Frequently asked questions
Links to more detailed information
So where should you start? Begin by reviewing this guide in its entirety and note those sections that you need
to revisit. Then you can create your own personalized study plan and schedule based on your individual needs
and how much time you have before test day.
Keep in mind that study habits are individual. There are many different ways to successfully prepare for your
test. Some people study better on their own, while others prefer a group dynamic. You may have more energy
early in the day, but another test taker may concentrate better in the evening. So use this guide to develop the
approach that works best for you.
Your teaching career begins with preparation. Good luck!
Know What to Expect
Which tests should I take?
Each state or agency that uses the Praxis tests sets its own requirements for which test or tests you must take for
the teaching area you wish to pursue.
Before you register for a test, confirm your state or agencys testing requirements at www.ets.org/praxis/states.
How are the Praxis tests given?
Praxis tests are given on computer. Other formats are available for test takers approved for accommodations (see
page 55).
The Praxis® Study Companion 3
Welcome to the Praxis® Study Companion
What should I expect when taking the test on computer?
When taking the test on computer, you can expect to be asked to provide proper identification at the test
center. Once admitted, you will be given the opportunity to learn how the computer interface works (how to
answer questions, how to skip questions, how to go back to questions you skipped, etc.) before the testing time
begins. Watch the What to Expect on Test Day video to see what the experience is like.
Where and when are the Praxis tests oered?
You can select the test center that is most convenient for you. The Praxis tests are administered through an
international network of test centers, which includes Prometric® Testing Centers, some universities, and other
locations throughout the world.
Testing schedules may differ, so see the Praxis Web site for more detailed test registration information at www.
ets.org/praxis/register.
The Praxis® Study Companion 4
Table of Contents
Table of Contents
The Praxis® Study Companion guides you through the steps to success
1. Learn About Your Test ....................................................................................................5
Learn about the specic test you will be taking
2. Familiarize Yourself with Test Questions ................................................................... 16
Become comfortable with the types of questions you’ll nd on the Praxis tests
3. Practice with Sample Test Questions ......................................................................... 20
Answer practice questions and nd explanations for correct answers
4. Determine Your Strategy for Success ......................................................................... 29
Set clear goals and deadlines so your test preparation is focused and ecient
5. Develop Your Study Plan ............................................................................................. 32
Develop a personalized study plan and schedule
6. Review Study Topics .................................................................................................... 36
Detailed study topics with questions for discussion
7. Review Smart Tips for Success .................................................................................... 53
Follow test-taking tips developed by experts
8. Check on Testing Accommodations ........................................................................... 55
See if you qualify for accommodations that may make it easier to take the Praxis test
9. Do Your Best on Test Day ............................................................................................. 56
Get ready for test day so you will be calm and condent
10. Understand Your Scores ............................................................................................ 58
Understand how tests are scored and how to interpret your test scores
Appendix: Other Questions You May Have ................................................................... 60
The Praxis® Study Companion 5
Step 1: Learn About Your Test
1. Learn About Your Test
Learn about the specic test you will be taking
Elementary Education: Content Knowledge (5018)
Test at a Glance
Test Name Elementary Education: Content Knowledge
Test Code 5018
Time 150 minutes
Number of Questions 140
Format Selected-response and numeric-entry questions, scientific calculator provided
Test Delivery Computer delivered
Approximate Approximate
Content Categories Number of Percentage of
Questions Examination
I. Reading and Language Arts 49 35%
II. Mathematics 41 29%
III. Social Studies 25 18%
IV. Science 25 18%
IV
III
II
I
About This Test
The Elementary Education: Content Knowledge test is designed for candidates who are preparing to enter
the field of elementary education. The test measures the knowledge, skills, and abilities judged by a national
advisory committee and a survey of education experts to be necessary for safe and effective practice. Test takers
are typically completing an undergraduate degree program in elementary education or have a degree in a
content area and are seeking an additional endorsement.
The 140 test questions focus on knowledge in four major content areas: reading and language arts,
mathematics, social studies, and science. Test takers are asked to show their knowledge of the topics covered
on the test in multiple ways using various types of responses: conceptual understanding, procedural awareness,
interpretation, integration, and application. The test is aligned with the Common Core State Standards for
English Language Arts and Mathematics, as well as with the content standards for each subject area.
An on-screen scientific calculator is provided for the computer-delivered test. Please consult the Praxis
Calculator Use web page for further information. You are expected to know how and when to use the scientific
calculator since it will be helpful for some questions. You are expected to become familiar with its functionality
before taking the test. To practice using the calculator, download the 30-day trial version and view tutorials
on how to use it. The calculator may be used to perform calculations, such as exponents, roots, and percents.
This test may contain some questions that will not count toward your score.
The Praxis® Study Companion 6
Step 1: Learn About Your Test
I. Reading and Language Arts
A. Reading: Foundational Skills
1. Understands the role of phonological
awareness in literacy development
a. Explains the importance of phonological
awareness as a foundational skill for literacy
development
b. Identifies and provides examples of
phonemes, syllables, onsets, and rimes
c. Identifies and provides examples of
blending, segmenting, substituting, and
deleting phonemes, syllables, onsets, rimes
2. Understands the role of phonics and word
analysis in literacy development
a. Explains the importance of phonics and
word analysis in literacy development
b. Distinguishes among common letter-sound
correspondences and spelling conventions
c. Distinguishes high-frequency sight words
from decodable words appropriate for
particular grades
d. Identifies roots and affixes to decode
unfamiliar words
e. Recognizes various stages of language
acquisition (e.g., WIDA taxonomy)
f. Delineates common phonics and word-
recognition approaches for ELLs (pedagogy)
g. Differentiates syllabication patterns (e.g.,
open, closed, CVe)
3. Understands the role of fluency (e.g., rate,
accuracy) in literacy development
a. Defines fluency and related terms (e.g.,
accuracy, rate, prosody)
b. Explains the impact of fluency on
comprehension
B. Reading: Literature and Informational Text
1. Understands how to use key ideas and details
to comprehend literature and informational text
a. Identifies the key details, moral, and/or
theme of a literary text, citing specific textual
evidence
b. Identifies the key details and/or central idea
of an informational text, citing specific
textual evidence
c. Makes inferences from a text and supports
them with appropriate evidence
d. Summarizes information from a text
e. Analyzes the characters, setting, and plot of
a literary text
f. Analyzes the relationships among
individuals, events, ideas, and concepts in an
informational text
2. Understands how features and structures of text
across genres affect comprehension
a. Identifies structural elements of literature
across genres (e.g., casts of characters and
stage directions in drama, rhyme and meter
in poetry)
b. Uses text features (e.g., headings, sidebars,
hyperlinks) to locate information in a print or
digital informational text
c. Identifies organizational structures of
informational text (e.g., cause/effect,
problem/solution)
d. Identifies how structural elements
contribute to the development of a literary
text as a whole
3. Understands the concept of point of view using
evidence from the text
a. Identifies authors point of view in various
genres and supports conclusions with
evidence from the text
b. Compares multiple accounts of the same
event or topic to identify similarities or
differences in point of view
c. Identifies how point of view impacts the
overall structure of a literary or informational
text
Test Specications
Test specifications in this chapter describe the knowledge and skills measured by the test. Study topics to help
you prepare to answer test questions can be found in page 36.
The Praxis® Study Companion 7
Step 1: Learn About Your Test
4. Understands how to integrate and compare
written, visual, and oral information from texts
and multimedia sources
a. Explains how visual and oral elements
enhance the meaning and effect of a literary
text (e.g., picture book, graphic novel,
multimedia presentation of a folktale)
b. Compares the written version of a literary
text with an oral, staged, or filmed version
c. Compares two or more literary texts that
address the same theme
d. Compares two or more informational texts
that address the same topic
e. Interprets visual and multimedia elements in
literary and informational texts
f. Evaluates key claims in a text and supports
them with reasons and evidence from the
text
5. Knows the role of text complexity in reading
development
a. Explains the three factors (i.e., quantitative,
qualitative, and reader and task) that
measure text complexity
b. Identifies features of text-leveling systems
C. Writing
1. Understands the characteristics of common
types of writing
a. Distinguishes among common types of
writing (e.g., opinion/argument, informative/
explanatory, narrative)
b. Identifies the purpose, key components, and
subgenres (e.g., speeches, advertisements,
narrative poems) of each common type of
writing
c. Evaluates the effectiveness of writing
samples of each type
2. Understands the characteristics of effective
writing
a. Evaluates the appropriateness of a particular
piece of writing for a specific task, purpose,
and audience
b. Evaluates the development, organization, or
style of a piece of writing
c. Identifies appropriate revisions to strengthen
a piece of writing
d. Writes clearly and coherently
e. Identifies the interrelationships among
planning, revising, and editing in the process
of writing
3. Knows the developmental stages of writing
(e.g., picture, scribble)
a. Identifies the grade-appropriate continuum
of student writing
4. Knows the importance of digital tools for
producing and publishing writing and for
interacting with others
a. Identifies the characteristics and purposes of
a variety of digital tools for producing and
publishing writing
b. Identifies the purposes of a variety of digital
tools for interacting with others
5. Knows the research process
a. Identifies the steps in the research process
b. Distinguishes between primary and
secondary sources and their uses
c. Distinguishes between reliable and
unreliable sources
d. Distinguishes between paraphrasing and
plagiarizing
e. Knows how to locate credible print and
digital sources, locate information within the
sources, and cite the sources
D. Language
1. Knows the conventions of standard English
grammar, usage, mechanics, and spelling when
writing, speaking, reading, and listening
a. Explains the function of different parts of
speech
b. Corrects errors in usage, mechanics, and
spelling
c. Identifies examples of different sentence
types (e.g., simple, compound, compound-
complex)
d. Identify how varieties of English (e.g.,
dialects, registers) used in stories, dramas, or
poems support the overall meaning
2. Understands how to determine the meaning of
words and phrases
a. Determines the literal meaning of unknown
words and phrases from context, syntax,
and/or knowledge of roots and affixes
b. Identifies types of figurative language
The Praxis® Study Companion 8
Step 1: Learn About Your Test
c. Interprets figurative language
d. Analyzes the relationship between word
choice and tone in a text
3. Understands characteristics of conversational,
academic, and domain-specific language
a. Differentiates among the three tiers of
vocabulary
b. Identifies relevant features of language such
as word choice, order, and punctuation
E. Speaking and Listening
1. Knows the characteristics of effective
collaboration to promote comprehension
a. Identifies techniques to communicate for a
variety of purposes with diverse partners
b. Identifies the characteristics of active
listening
2. Knows the characteristics of engaging oral
presentations
a. Identifies elements of engaging oral
presentations (e.g., volume, articulation,
awareness of audience)
II. Mathematics
A. Numbers and Operations
1. Understands the place value system
a. Writes numbers using base-10 numerals,
number names, and expanded form
b. Composes and decomposes multi-digit
numbers
c. Given a digit, identifies the place the digit is
in and its value in that place
d. Recognizes that a digit in one place
represents ten times what it represents in
the place to its right and one-tenth what it
represents in the place to its left, and extend
this recognition to several place to the right
or left
e. Uses whole-number exponents to denote
powers of 10
f. Rounds multi-digit numbers to any place
value
2. Understands operations and properties of
rational numbers
a. Solves multistep mathematical and real-
world problems using addition, subtraction,
multiplication, and division of rational
numbers and shows knowledge of how to
classify problem situations, inverse
operations, remainders, concepts of zero,
absolute value, and opposites
b. Understands various strategies and
algorithms used to perform operations on
rational numbers
c. Recognizes concepts of rational numbers
and their operations, including those related
to unit fractions, composition and
decomposition of fractions, comparing
fractions
d. Solves problems using the order of
operations, including problems involving
whole-number exponents
e. Identifies properties of operations (e.g.,
commutative, associative, distributive) and
uses them to solve problems
f. Represents rational numbers and their
operations in different ways, using drawings,
models, number lines, arrays
g. Compares, classifies, and orders rational
numbers
h. Converts between fractions, decimals, and
percents
3. Understands proportional relationships and
percents
a. Applies the concepts of ratios and unit rates
to describe relationships between two
quantities
b. Understands percent as a rate per 100
c. Solves unit-rate problems
d. Uses proportional relationships to solve ratio
and percent problems
4. Knows how to use basic concepts of number
theory
a. Identifies and uses prime and composite
numbers
b. Finds factors and multiples of numbers
The Praxis® Study Companion 9
Step 1: Learn About Your Test
5. Knows a variety of strategies to determine
reasonableness of results
a. Recognizes the reasonableness of results
within the context of a given problem
b. Uses mental math, estimation, and rounding
strategies to solve problems and determine
reasonableness of results
B. Algebraic Thinking
1. Knows how to evaluate and manipulate
algebraic expressions, equations, and formulas
a. Differentiates between algebraic expressions
and equations
b. Adds and subtracts linear algebraic
expressions
c. Uses the distributive property to generate
equivalent linear algebraic expressions
d. Evaluates simple algebraic expressions (i.e.,
one variable, binomial) for given values of
variables
e. Uses mathematical terms to identify parts of
expressions and describe expressions
f. Translates between verbal statements and
algebraic expressions or equations (e.g., the
phrase “the number of cookies Joe has is
equal to twice the number of cookies Sue
has” can be represented by the equation
j = 2s )
g. Uses formulas to determine unknown
quantities
h. Differentiates between dependent and
independent variables in formulas
2. Understands the meanings of the solutions to
linear equations and inequalities
a. Solves multistep one-variable linear
equations and inequalities
b. Interprets solutions of multistep one-
variable linear equations and inequalities
(e.g., graphs the solution on a number line,
states constraints on a situation)
c. Uses linear relationships represented by
equations, tables, and graphs to solve
problems
3. Knows how to recognize and represent patterns
(e.g., number, shape)
a. Identifies, extends, describes, or generates
number and shape patterns
b. Makes conjectures, predictions, or
generalizations based on patterns
c. Identifies relationships between the
corresponding terms of two numerical
patterns (e.g., find a rule for a function table)
C. Geometry and Measurement
1. Understands how to classify one-, two-, and
three-dimensional figures
a. Uses definitions to identify lines, rays, line
segments, parallel lines, and perpendicular
lines
b. Classifies angles based on their measure
c. Composes and decomposes two- and three-
dimensional shapes
d. Uses attributes to classify or draw polygons
and solids
2. Knows how to solve problems involving
perimeter, area, surface area, and volume
a. Represents three-dimensional figures with
nets
b. Use nets that are made of rectangles and
triangles to determine the surface area of
three-dimensional figures
c. Finds the area and perimeter of polygons,
including those with fractional side lengths
d. Finds the volume and surface area of right
rectangular prisms, including those with
fractional edge lengths
e. Determines how changes to dimensions
change area and volume
3. Knows the components of the coordinate plane
and how to graph ordered pairs on the plane
a. Identifies the x-axis, the y-axis, the origin,
and the four quadrants in the coordinate
plane
b. Solves problems by plotting points and
drawing polygons in the coordinate plane
4. Knows how to solve problems involving
measurement
a. Solves problems involving elapsed time,
money, length, volume, and mass
b. Measures and compares lengths of objects
using standard tools
c. Knows relative sizes of United States
customary units and metric units
d. Converts units within both the United States
customary system and the metric system
The Praxis® Study Companion 10
Step 1: Learn About Your Test
D. Data, Statistics, and Probability
1. Is familiar with basic statistical concepts
a. Identifies statistical questions
b. Solves problems involving measures of
center (mean, median, mode) and range
c. Recognizes which measure of center best
describes a set of data
d. Determines how changes in data affect
measures of center or range
e. Describes a set of data (e.g., overall patterns,
outliers)
2. Knows how to represent and interpret data
presented in various forms
a. Interprets various displays of data (e.g., box
plots, histograms, scatterplots)
b. Identifies, constructs, and completes graphs
that correctly represent given data (e.g.,
circle graphs, bar graphs, line graphs,
histograms, scatterplots, double bar graphs,
double line graphs, box plots, and line plots/
dot plots)
c. Chooses appropriate graphs to display data
3. Is familiar with how to interpret the probability
of events
a. Interprets probabilities relative to likelihood
of occurrence
III. Social Studies
A. Geography, Anthropology, and Sociology
1. Knows world and regional geography
a. Is familiar with spatial terms and can identify
spatial patterns of people, places, and
environments
b. Identifies the characteristics of places and
regions
c. Locates major physical features of
geography (e.g., mountain ranges, bodies of
water)
d. Locates major political features of geography
(e.g., continents, countries, states, cities)
e. Demonstrates basic geographic literacy (e.g.,
uses and interpretations of different types of
maps, understanding of the concepts of
absolute and relative location, identification
of cardinal and intermediate directions)
2. Understands the interaction of physical and
human systems
a. Demonstrates knowledge of how humans
change the environment
b. Demonstrates knowledge of how the
environment affects human activities
c. Understands the importance of natural and
human resources
3. Knows the uses of geography
a. Applies geography to interpret the past and
the present and to plan for the future
4. Knows how people of different backgrounds
interact with the their environment, self, family,
neighborhood, and organizations.
a. Demonstrates knowledge of how human
behavior is influenced by society and by
societys groups, institutions, and
organizations.
B. World History
1. Knows the major contributions of classical
civilizations such as Egypt, Greece, and Rome
a. Demonstrates knowledge of how modern
civilizations reflect, mirror, and learn from the
contributions of ancient civilizations
2. Understands twentieth-century developments
and transformations in world history
a. Demonstrates knowledge of the causes and
effects of the First and Second World Wars
and the Cold War
b. Demonstrates knowledge of technological
developments (e.g., transportation,
communication, tools)
c. Demonstrates knowledge of the causes and
effects of globalization
3. Understands the role of cross-cultural
comparisons in world history instruction
a. Demonstrates knowledge of various
psychological, sociological, and cultural
factors needed to assess the similarities and/
or diversities in two or more different
cultures or societies
The Praxis® Study Companion 11
Step 1: Learn About Your Test
C. United States History
1. Knows about the European exploration and
colonization of North America and growth and
expansion of the United States
a. Demonstrates knowledge of Native
American peoples and cultures
b. Demonstrates knowledge of the reasons for
the colonization of North America and the
development of the thirteen colonies
c. Is familiar with the interactions between
Native American groups, colonists, and
European powers
2. Knows about the American Revolution and the
founding of the United States
a. Understands the causes and effects of the
American Revolution
b. Identifies key individuals and events during
the American Revolution
c. Demonstrates knowledge of the challenges
faced by the early republic (e.g., creation of a
democratic government)
3. Knows about the major events and
developments in United States history from
founding to present
a. Demonstrates knowledge of the causes and
effects of the territorial expansion of the
United States (e.g., concept of Manifest
Destiny; Louisiana Purchase; impact on
Native Americans; role of technological,
political, and economic developments)
b. Understands the causes and effects of the
Civil War (e.g., growth of sectionalism, the
abolition movement, the Underground
Railroad, the reasons for the succession of
the Confederate States, the role of Abraham
Lincoln, the purposes and challenges of
Reconstruction)
c. Demonstrates knowledge of the causes and
effects of industrialization, urbanization, and
immigration
d. Is familiar with major social and cultural
developments throughout United States
history
4. Knows about twentieth-century developments
and transformations in the United States
a. Demonstrates knowledge of the causes and
effects of the Great Depression (e.g., New
Deal legislation)
b. Demonstrates knowledge of the causes and
effects of the First and Second World Wars
and the Cold War
c. Demonstrates knowledge of major
economic developments (e.g., assembly line,
mass production,) and the influence of
technological developments
5. Understands connections between the causes
and effects of events
a. Demonstrates the ability to draw
connections between the causes and effects
of significant events throughout United
States history
D. Government, Citizenship, and Democracy
1. Understands the nature, purpose, and forms of
government
a. Is familiar with the founding principles of the
United States government (e.g.,
republicanism, separation of powers, checks
and balances, popular sovereignty)
b. Demonstrates knowledge of federalism (e.g.,
division of power between the national and
state governments)
c. Demonstrates knowledge of the powers of
the three branches of the federal
government and the interactions among
them
d. Is familiar with basic characteristics of
different political systems
2. Knows key documents and speeches in the
history of the United States
a. Is familiar with the purpose and contents of
the Declaration of Independence
b. Is familiar with the Articles of Confederation
c. Demonstrates knowledge of the structure of
government outlined in the United States
Constitution
d. Demonstrates knowledge of the rights and
protections guaranteed to United States
citizens by the Constitution
e. Is familiar with key documents and speeches
(e.g., Gettysburg Address)
The Praxis® Study Companion 12
Step 1: Learn About Your Test
3. Knows the rights and responsibilities of
citizenship in a democracy
a. Demonstrates knowledge of civic
participation (e.g., community service,
membership in civic organizations)
b. Demonstrates knowledge of the rights and
responsibilities of citizens in the United
States (e.g. voting, paying taxes, freedom of
speech)
E. Economics
1. Knows key terms and basic concepts of
economics
a. Demonstrates knowledge of supply and
demand
b. Is familiar with concepts of scarcity, choice,
and opportunity cost
c. Demonstrates knowledge of the role of
money and resources in economic decision
making
2. Understands how economics affects
population, resources, and technology
a. Demonstrates an understanding of how
people use resources to generate wealth
and enhance their lives
b. Demonstrates an understanding of how
economics drives and is driven by
technological innovations
3. Understands the government’s role in
economics and the impact of economics on
government
a. Demonstrates knowledge of the federal
government’s role in regulating the
economy
b. Demonstrates knowledge of taxing and
spending
F. Social Studies as Inquiry and Social Studies
Processes
1. Understands social studies as inquiry
a. Demonstrates knowledge of questioning,
gathering data, and drawing reasonable
conclusions
2. Understands how to use resource and research
material in social studies
a. Understands how to evaluate the
appropriate uses of a variety of resources
b. Identifies primary and secondary sources
and demonstrates knowledge of the uses of
each
c. Demonstrates knowledge of fact and
opinion and knows the uses of each in social
studies
3. Understands process skills in social studies
a. Understands how to interpret different types
of information
b. Evaluates relationships among different
variables
c. Demonstrates ability to draw conclusions
using tools of the field
IV. Science
A. Earth and Space Science
1. Understands basic physical and historical
geology
a. Identify Earths basic structure (e.g., mantle,
core, geographical features such as
mountains, magnetic field)
b. Identify and describe types and
characteristics of rocks and minerals
c. Recognize processes involved in erosion,
weathering, and deposition of Earths surface
materials
d. Recognize Earths internal processes
including impact of plate tectonic theory
(e.g., volcanoes, earthquakes)
e. Identify key aspects of the water cycle (e.g.,
evaporation, condensation, precipitation,
runoff )
f. Recognize important events in Earths
geologic history and the importance of the
rock record and fossils
2. Is familiar with the structure and processes of
Earths hydrosphere
a. Identify the geographic location of Earths
oceans and seas and the processes involved
with tides and waves
b. Identify characteristics of lakes, streams
rivers, polar ice, icebergs, glaciers, and
groundwater
c. Identify the basic characteristics of Earth’s
atmosphere
d. Recognize the basic concepts of weather
(e.g., clouds, precipitation, hurricanes)
e. Identify factors that affect climate and
seasons (e.g., climate zones, proximity to
mountains and oceans)
The Praxis® Study Companion 13
Step 1: Learn About Your Test
3. Is familiar with astronomy
a. Identify the major features of the solar
system, including the Sun, the planets,
moons, asteroids, and comets
b. Recognize the interactions of the Earth-
Moon-Sun system (e.g., phases of the Moon,
eclipses, seasons, tides
c. Recognize the major features of the universe
a (e.g., galaxies, stars, black holes)
B. Life Sciences
1. Understands the basic structure and function of
cells and levels of organization in living things
a. Identify the structure and function of cell
organelles (e.g., nucleus, cell membrane)
b. Recognize basic cell processes such as cell
division and photosynthesis
c. Identify the levels of organization (cells,
tissues, organs, organ systems)
2. Understands basic genetics and evolution
a. Apply basic genetics (e.g., relationship
between genes and traits)
b. Recognize the basic structure and function
of DNA and relationship to heredity
c. Recognize common human genetic
disorders
d. Identify processes by which species change
over time, including natural selection,
mutation, evolution
3. Knows the hierarchical classification scheme
and the characteristics of the major groups of
organisms
a. Identify elements of classification schemes
(e.g., kingdom, genus, species)
b. Identify major characteristics of common
types of organisms (e.g. amphibians, reptiles,
mammals, plants)
4. Knows the major structures and functions of
plant organs and systems
a. Identify the basic structure and function of
leaves, roots, and stems
b. Recognize key aspects of asexual and sexual
reproduction, development, and growth
c. Recognize the uptake and transport of
nutrients and water
5. Knows the basic anatomy and physiology of
animals, including human body systems
a. Identify examples of exchange with the
environment involving the respiratory,
excretory, and digestive systems
b. Recognize key aspects of internal transport
and exchange in terms of the circulatory
system
c. Recognize key aspects of support and
movement in terms of the skeletal and
muscular systems
d. Identify key aspects of reproduction and
development
e. Recognize the function of immune systems
f. Identify the functions of immune systems,
nervous systems, and endocrine systems
g. Recognize the importance of homeostasis
6. Knows key aspects of ecology
a. Recognize key relationships between and
among species such as territoriality,
predator-prey, and parasitism
b. Recognize key aspects of ecosystems (e.g.,
biomes, energy levels, food webs, effect of
disturbances)
C. Physical Sciences
1. Knows the basic structure and properties of
matter
a. Identify basic properties of solids, liquids and
gases (e.g., structure, density, conductivity,
solubility)
b. Identify and distinguish between elements,
atoms, compounds, molecules, and mixtures
c. Describe the atomic model, including
electrons, protons, neutrons, atomic number
and atomic mass
d. Is familiar with the periodic table of the
elements, its symbols and the information it
provides
2. Knows the basic relationships between energy
and matter
a. Recognize that energy and matter is
conserved in various situations
b. Recognize how various forms of kinetic and
potential energy can be transformed from
one form to another
c. Identify the differences between chemical
and physical properties/changes
The Praxis® Study Companion 14
Step 1: Learn About Your Test
d. Describe methods of heat transfer
(convection, radiation, conduction)
e. Describe how the states of matter undergo
phase changes and the energy changes
involved
3. Understands basic chemical reactions
a. Identify the difference between covalent
and ionic bonding
b. Interpret simple chemical formulas
c. Recognize that chemical reactions involve
energy changes
d. Identify chemical and physical properties of
acids and bases and the pH scale
e. Recognize common types of chemical
reactions such as neutralization, oxidation,
and combustion
4. Understands basic concepts in mechanics
a. Describe motion in terms of distance, speed,
velocity, and acceleration
b. Describe the effect of forces on objects (e.g.,
collisions, pendulums, friction)
c. Recognize the effect of gravity and
distinguish between mass and weight
d. Recognize forces and physical properties
involving fluids that determine whether
objects will sink or float
5. Understands basic concepts in electricity,
magnetism, waves, and optics
a. Describe basic characteristics of magnets
(e.g., magnetic poles, attraction, repulsion)
b. Recognize electrostatic attraction and
repulsion
c. Describe electricity in terms of the flow of
electrons and identify voltage sources
(batteries and generators)
d. Describe the basic phenomena involving
light (reflection, rainbows, mirrors, prisms)
e. Describe basic characteristics of sound
(pitch, loudness, the Doppler effect)
D. Impact of Science and Technology on Society
1. Knows the impact of science and technology
on the environment and society
a. Recognize the impact of air and water
pollution, greenhouse gases
b. Recognize the impact of production and
disposal of consumer products
c. Recognize the benefits of conservation and
recycling
d. Identify renewable and nonrenewable
energy resources
e. Identify the pros and cons of power
generation based on various sources (e.g.,
fossil, nuclear, water, wind, solar, biomass,
geothermal)
2. Is familiar with applications of science and
technology in daily life and public health
a. Identify applications of chemical and
physical principles related to common
consumer products (e.g., acid-base
properties of orange juice, applications of
physics in devices such as lenses)
b. Identify common agricultural practices (e.g.,
genetically modified crops, use of herbicides
and insecticides)
c. Recognize the role of nutrition, disease, and
medicine (e.g., food preservation, vitamins,
vaccines, viruses)
d. Recognize applications of medical
technologies (e.g., MRIs, X-rays, radiation
therapy)
E. Science as Inquiry and Science Processes
1. Understands the basic elements of scientific
inquiry and how they are used
a. Identify hypotheses, theories, models, and
laws, and their role in scientific inquiry
b. Explain the role of the elements of
experimental design, including independent
and dependent variables, controls, sources
of error, and drawing conclusions
c. Recognize that scientific knowledge is
subject to change, consistent with evidence,
based on reproducible evidence and
includes unifying concepts and processes
(e.g., systems, models, constancy and
change, equilibrium, form and function)
d. Recognize how key concepts developed
over time and identify the contribution of
key historical figures (e.g., Newtons laws,
Marie Curie’s work with radioactivity,
Mendel’s development of basic genetics)
The Praxis® Study Companion 15
Step 1: Learn About Your Test
2. Understands the common methods and tools
used to gather and present reliable data
a. Identify common units of measurement
(e.g., meter, gram, liter)
b. Explain the appropriate use of common
measurement tools (e.g., thermometers,
barometers, balances)
c. Organize and present data (e.g., graphs,
tables, charts, maps)
3. Knows how to interpret and draw conclusions
from data presented in tables, graphs, charts,
and maps
a. Identify patterns and significant points in
data
b. Draw conclusions and make predictions
based on presented data
c. Recognize relationships between variables
d. Recognize the effect of error on data and
conclusions
4. Understands procedures for safe and correct
use of laboratory materials and equipment
a. Recognize safe and appropriate methods to
prepare materials for classroom use
(activities and demonstrations)
b. Recognize when and how to use standard
equipment in the laboratory (e.g.,
microscopes, graduated cylinders)
c. Explain the use of standard safety
equipment (e.g., eyewash stations, safety
showers)
d. Identify appropriate student apparel and
behavior (e.g., goggles, clothing, no eating in
lab)
e. Recognize emergency procedures for
mishaps (e.g., fires, chemical spills, injuries)
and evacuation procedures
The Praxis® Study Companion 16
Step 2: Familiarize Yourself with Test Questions
2. Familiarize Yourself with Test Questions
Become comfortable with the types of questions you’ll nd on the Praxis tests
The Praxis Series assessments include a variety of question types: constructed response (for which you write a
response of your own); selected response, for which you select one or more answers from a list of choices or
make another kind of selection (e.g., by clicking on a sentence in a text or by clicking on part of a graphic); and
numeric entry, for which you enter a numeric value in an answer field. You may be familiar with these question
formats from taking other standardized tests. If not, familiarize yourself with them so you don’t spend time
during the test figuring out how to answer them.
Understanding Computer-Delivered Questions
Questions on computer-delivered tests are interactive in the sense that you answer by selecting an option
or entering text on the screen. If you see a format you are not familiar with, read the directions carefully. The
directions always give clear instructions on how you are expected to respond.
For most questions, you respond by clicking an oval to select a single answer from a list of options.
However, interactive question types may also ask you to respond by:
Clicking more than one oval to select answers from a list of options.
Typing in an entry box. When the answer is a number, you may be asked to enter a numerical answer.
Some questions may have more than one place to enter a response.
Clicking check boxes. You may be asked to click check boxes instead of an oval when more than one
choice within a set of answers can be selected.
Clicking parts of a graphic. In some questions, you will select your answers by clicking on a location (or
locations) on a graphic such as a map or chart, as opposed to choosing your answer from a list.
Clicking on sentences. In questions with reading passages, you may be asked to choose your answers by
clicking on a sentence (or sentences) within the reading passage.
Dragging and dropping answer choices into targets on the screen. You may be asked to select answers
from a list of options and drag your answers to the appropriate location in a table, paragraph of text or
graphic.
Selecting options from a drop-down menu. You may be asked to choose answers by selecting options
from a drop-down menu (e.g., to complete a sentence).
Remember that with every question you will get clear instructions.
Perhaps the best way to understand computer-delivered questions is to view the Computer-delivered Testing
Demonstration on the Praxis Web site to learn how a computer-delivered test works and see examples of
some types of questions you may encounter.
The Praxis® Study Companion 17
Step 2: Familiarize Yourself with Test Questions
Understanding Selected-Response Questions
Many selected-response questions begin with the phrase which of the following. Take a look at this example:
Which of the following is a avor made from beans?
(A) Strawberry
(B) Cherry
(C) Vanilla
(D) Mint
How would you answer this question?
All of the answer choices are flavors. Your job is to decide which of the flavors is the one made from beans.
Try following these steps to select the correct answer.
1) Limit your answer to the choices given. You may know that chocolate and coffee are also flavors made
from beans, but they are not listed. Rather than thinking of other possible answers, focus only on the choices
given (“which of the following”).
2) Eliminate incorrect answers. You may know that strawberry and cherry flavors are made from fruit and
that mint flavor is made from a plant. That leaves vanilla as the only possible answer.
3) Verify your answer. You can substitute “vanilla for the phrase “which of the following and turn the
question into this statement: Vanilla is a flavor made from beans. This will help you be sure that your answer
is correct. If you’re still uncertain, try substituting the other choices to see if they make sense. You may want
to use this technique as you answer selected-response questions on the practice tests.
Try a more challenging example
The vanilla bean question is pretty straightforward, but you’ll find that more challenging questions have a
similar structure. For example:
Entries in outlines are generally arranged according
to which of the following relationships of ideas?
(A) Literal and inferential
(B) Concrete and abstract
(C) Linear and recursive
(D) Main and subordinate
You’ll notice that this example also contains the phrase “which of the following. This phrase helps you
determine that your answer will be a relationship of ideas from the choices provided. You are supposed to find
the choice that describes how entries, or ideas, in outlines are related.
Sometimes it helps to put the question in your own words. Here, you could paraphrase the question in this way:
“How are outlines usually organized?” Since the ideas in outlines usually appear as main ideas and subordinate
ideas, the answer is (D).
The Praxis® Study Companion 18
Step 2: Familiarize Yourself with Test Questions
QUICK TIP: Don’t be intimidated by words you may not understand. It might be easy to be thrown by words
like “recursive or “inferential. Read carefully to understand the question and look for an answer that fits. An
outline is something you are probably familiar with and expect to teach to your students. So slow down, and
use what you know.
Watch out for selected-response questions containing “NOT, “LEAST, and “EXCEPT”
This type of question asks you to select the choice that does not fit. You must be very careful because it is easy
to forget that you are selecting the negative. This question type is used in situations in which there are several
good solutions or ways to approach something, but also a clearly wrong way.
How to approach questions about graphs, tables, or reading passages
When answering questions about graphs, tables, or reading passages, provide only the information that the
questions ask for. In the case of a map or graph, you might want to read the questions first, and then look at the
map or graph. In the case of a long reading passage, you might want to go ahead and read the passage first,
noting places you think are important, and then answer the questions. Again, the important thing is to be sure
you answer the questions as they refer to the material presented. So read the questions carefully.
How to approach unfamiliar formats
New question formats are developed from time to time to find new ways of assessing knowledge. Tests may
include audio and video components, such as a movie clip or animation, instead of a map or reading passage.
Other tests may allow you to zoom in on details in a graphic or picture.
Tests may also include interactive questions. These questions take advantage of technology to assess
knowledge and skills in ways that standard selected-response questions cannot. If you see a format you are
not familiar with, read the directions carefully. The directions always give clear instructions on how you are
expected to respond.
QUICK TIP: Don’t make the questions more difficult than they are. Don’t read for hidden meanings or tricks.
There are no trick questions on Praxis tests. They are intended to be serious, straightforward tests of
your knowledge.
Understanding Constructed-Response Questions
Constructed-response questions require you to demonstrate your knowledge in a subject area by creating
your own response to particular topics. Essays and short-answer questions are types of constructed-response
questions.
For example, an essay question might present you with a topic and ask you to discuss the extent to which you
agree or disagree with the opinion stated. You must support your position with specific reasons and examples
from your own experience, observations, or reading.
Take a look at a few sample essay topics:
“Celebrities have a tremendous influence on the young, and for that reason, they have a responsibility to
act as role models.
We are constantly bombarded by advertisements—on television and radio, in newspapers and
magazines, on highway signs, and the sides of buses. They have become too pervasive. Its time to put
limits on advertising.
Advances in computer technology have made the classroom unnecessary, since students and teachers
are able to communicate with one another from computer terminals at home or at work.
The Praxis® Study Companion 19
Step 2: Familiarize Yourself with Test Questions
Keep these things in mind when you respond to a constructed-response question
1) Answer the question accurately. Analyze what each part of the question is asking you to do. If the
question asks you to describe or discuss, you should provide more than just a list.
2) Answer the question completely. If a question asks you to do three distinct things in your response,
you should cover all three things for the best score. Otherwise, no matter how well you write, you will
not be awarded full credit.
3) Answer the question that is asked. Do not change the question or challenge the basis of the
question. You will receive no credit or a low score if you answer another question or if you state, for
example, that there is no possible answer.
4) Give a thorough and detailed response. You must demonstrate that you have a thorough
understanding of the subject matter. However, your response should be straightforward and not filled
with unnecessary information.
5) Reread your response. Check that you have written what you thought you wrote. Be sure not to
leave sentences unfinished or omit clarifying information.
QUICK TIP: You may find that it helps to take notes on scratch paper so that you don’t miss any details. Then
you’ll be sure to have all the information you need to answer the question.
For tests that have constructed-response questions, more detailed information can be found on page 5.
The Praxis® Study Companion 20
Step 3: Practice with Sample Test Questions
3. Practice with Sample Test Questions
Answer practice questions and nd explanations for correct answers
Sample Test Questions
This test is available on computer. The following sample question provides a preview of the actual screen used
in a computer-delivered test. For the purposes of this Study Companion, the sample questions in this chapter
are shown as they would appear in a paper-delivered test.
The Praxis® Study Companion 21
Step 3: Practice with Sample Test Questions
The sample questions that follow illustrate the kinds of
questions on the test. They are not, however, representative
of the entire scope of the test in either content or difficulty.
Answers with explanations follow the questions.
Directions: Each of the questions or statements below
is followed by suggested answers or completions. Select
the answer or answers that are best in each case.
I. Language Arts
1. How many phonemes are in the word “ball”?
(A) 1
(B) 2
(C) 3
(D) 4
2. Which TWO of the following syllable types are
present in the word “remarkable”?
(A) Open
(B) Closed
(C) Vowel team
(D) R-controlled
(E) Vowel-consonant-e
3. Which of the following is the most appropriate
strategy for using easy books to increase
uency in a nonuent student?
(A) Providing opportunities for the nonuent
student to read self-selected easy books
to a younger student
(B) Asking a younger, more uent reader to
read an easy book aloud to the nonuent
student
(C) Assigning an easy-to-read nonction
book to the nonuent student for
independent reading
(D) Encouraging other students to interrupt
and correct when the nonuent student
is reading easy books aloud
4. Which of the following is most commonly
used in digital text to give a reader access to
additional information about a topic?
(A) A citation
(B) A hyperlink
(C) An index
(D) A glossary
5. Which of the following statements best
describes how graphic novels promote
inferencing?
(A) Readers rely on characters’ dialogue to
tell the story.
(B) Readers are given graphic organizers to
facilitate understanding.
(C) Readers can summarize the stories’
beginning, middle, and end.
(D) Readers use the pictures to interpret the
text.
6. Which of the following is true of qualitative
measures of text complexity?
(A) They describe statistical measurements
of a text.
(B) They rely on computer algorithms to
describe text.
(C) They involve attributes that can be
measured only by human readers.
(D) They account for the different
motivational levels readers bring to texts.
7. Which TWO of the following can be classied
as expository writing?
(A) A short story
(B) A technical speech
(C) A personal diary
(D) A scientic report
(E) An editorial commentary
8. Which of the following technology-based
tools best facilitates both personal writing and
written discussion about the writing?
(A) Blogging programs
(B) Interactive gaming
(C) Slide-share programs
(D) Interactive whiteboards
The Praxis® Study Companion 22
Step 3: Practice with Sample Test Questions
Questions 9-10 refer to the following sonnet.
The following is an excerpt from Sir Philip Sidney’s
sonnet “Leave me, O Love, which reachest but to
dust.”
Leave me, O Love, which reachest but to dust;
And thou, my mind, aspire to higher things;
Grow rich in that which never taketh rust;
Whatever fades but fading pleasure brings.
9. In line 1, “dust” serves as a metaphor for
(A) ignorance
(B) death
(C) loneliness
(D) confusion
10. The lines comment on the speaker’s desire to
(A) seek out immediate pleasures
(B) enrich himself
(C) reject that which is transitory
(D) revive the past
11. Two hamsters sat in the cage side by side; a
furtive, timid one and a glossy, bold one
watched each other warily.
The sentence above is an example of a
(A) simple sentence
(B) compound sentence
(C) complex sentence
(D) compound-complex sentence
12. Which of the following is most typically
included in the conclusion of an oral
presentation?
(A) An expansion of the thesis
(B) A summarization of the main points
(C) An attempt to build rapport with the
audience
(D) A move to gain the audience’s attention
13. In a student discussion about whether the
school cafeteria should stop selling junk food,
which of the following statements best
demonstrates active listening?
(A) “In my opinion, it would be a mistake to
remove junk food from the cafeteria
because no one would eat there
anymore.”
(B) “Raul thinks that our health should come
before eating what we love, but Lacey
argues that the schools should not take
away our right to choose.”
(C) “How many of you would actually buy
lunch if the cafeteria stopped selling junk
food?”
(D) “What if we write a formal complaint to
the superintendent to voice our opinion
on the food in the cafeteria?”
II. Mathematics
14. Which of the following is an example of the
commutative property of addition?
(A) 5 x 3 = 3 x 5
(B) (1 + 7) + 4 = 1 + (7 + 4)
(C) 6 x (4 + 2) = (6 x 4) + (6 x 2)
(D) 8 + 9 = 9 + 8
x y
420
738
12 68
15 86
15. There is a linear relationship between x and y
in the table above. Which of the following
equations gives the rule that relates x and y ?
(A) y = 8x + 4
(B) y = 7x + 4
(C) y = 6x + 4
(D) y = 5x + 4
The Praxis® Study Companion 23
Step 3: Practice with Sample Test Questions
16. A fourth-grade class started working on math
worksheets at 1:30 p.m. and stopped working
at 3:10 p.m. How long did the class work on
the math worksheets?
(A) 40 minutes
(B) 80 minutes
(C) 100 minutes
(D) 120 minutes
17. A student plans to simultaneously toss a fair
number cube, with faces numbered 1 through
6, and a fair coin. What is the probability that
the cube will land with the face numbered 4
up and the coin will land heads up?
(A)
1
12
(B) 1
8
(C)
1
6
(D) 2
3
18. The only prime factors of a certain number are
2, 3, and 7. Which of the following could be
the number?
(A) 18 x 28
(B) 20 x 21
(C) 22 x 63
(D) 24 x 35
57≤−p
19. Which of the following is equivalent to the
inequality above?
(A) p
2
(B) p
2
(C) p
≤−
2
(D) p
≥−
2
20. Which of the following best describes the
polygon above?
(A) A regular hexagon
(B) An arrow
(C) A convex hexagon
(D) A concave hexagon
21. After a lesson on rounding and estimation, a
teacher tells students that a cook has made
157 sandwiches for 4 lunch periods. The
teacher asks the students to estimate the
average number of sandwiches that will be
served in each lunch period. Which of the
following students correctly estimated the
answer?
(A) Student A: about 35
(B) Student B: 39.25
(C) Student C: about 40
(D) Student D: about 45
22. Jack had three babysitting jobs this week. He
worked the same number of hours H on each
job. He was paid $12 per hour at his rst job,
$4 per half hour at his second job, and $5 for
each 20 minutes at his third job. Which of the
following expressions could be used to nd
the total amount, in dollars, Jack earned?
(A)
1245×+×+×HHH
(B)
12 815×+×+ ×HH H
(C)
12820×+×+ ×HH H
(D) 12 41
51
×+×××HHH
The Praxis® Study Companion 24
Step 3: Practice with Sample Test Questions
23. To make fruit punch, Edie mixes two kinds of
juices in the following ratio: 1 cup of blueberry
to 3 cups of red raspberry. How many cups of
red raspberry will Edie need to make 48 cups
of fruit punch?
(A) 12
(B) 16
(C) 24
(D) 36
III. Social Studies
24. Mount Rainier is located in which of the
following mountain ranges?
(A) The Cascades
(B) The Rockies
(C) The Appalachians
(D) The Alleghenies
25. Which of the following is believed to have
occurred during the last Ice Age as a result of
a land bridge created between what are now
Siberia and Alaska?
(A) The invention of new technologies for
sheltering humans against sustained
cold
(B) The blockage of important trade routes
(C) The establishment of human settlements
in North America
(D) Widespread famine
26. Since the end of the United States Civil War in
1865, all of the following have been major
objectives of groups seeking civil rights for
Black people except
(A) passage of afrmative action legislation
(B) desegregation of public educational
facilities
(C) creation of a third party in national
politics
(D) passage of anti-lynching laws
27. What percent of the seats in the United States
House of Representatives are up for election
every two years?
(A) 33%
(B) 50%
(C) 66%
(D) 100%
28. Historically, India’s society has been
organized into hierarchical groups known as
(A) tribes
(B) castes
(C) clans
(D) denominations
29. According to the graph above, how many of
the countries shown produced more crude oil
in 1975 than 1974?
(A) 1
(B) 2
(C) 3
(D) 4
The Praxis® Study Companion 25
Step 3: Practice with Sample Test Questions
IV. Science
30. Which of the following geological processes
adds new rock to the surface of Earth?
(A) Volcanic activity
(B) Glacial activity
(C) Soil erosion
(D) Weathering
31. Which of the diagrams above best depicts the
Moon as viewed from Earth at the rst quarter
of the lunar cycle?
(A) 1
(B) 2
(C) 3
(D) 4
32. Which of the following is NOT a way in which
mammals keep themselves warm in winter?
(A) Shivering
(B) Perspiring
(C) Flufng out coat hair
(D) Contracting certain blood vessels
33. Which of the following would be observed in a
vacuum if a feather and two stones of
different weights were dropped
simultaneously from a height of ten feet?
(A) Both stones would hit the ground at the
same time, but before the feather.
(B) The heavier stone would hit the ground
rst.
(C) The lighter stone would hit the ground
rst.
(D) All three objects would hit the ground at
the same time.
34. Which of the following laboratory instruments
would be most appropriate to use in
determining the volume of a large block of
wood of unknown density?
(A) A metric ruler
(B) A triple-beam balance
(C) A 200 mL volumetric ask
(D) A micrometer
35. Which of the following best describes a
scientic hypothesis?
(A) It ensures that successful results will be
obtained from an experiment.
(B) It must be accepted as true by the
scientic community.
(C) It is a testable proposal that may lead to
experimentation.
(D) It must be formulated by a renowned
scientist.
36. Which of the following is the broadest
category in the biological taxonomy?
(A) Kingdom
(B) Order
(C) Genus
(D) Species
The Praxis® Study Companion 26
Step 3: Practice with Sample Test Questions
1. The correct answer is (C). The word structure is
/b/ /a/ /l/. “ll is a blend so therefore makes only one
sound
2. The correct answers are (A) and (D). (A) is correct
because the syllable “re is an open syllable. An open
syllable ends in a long vowel sound produced by a
single vowel. (D) is correct because the syllable “mar is
an r-controlled syllable. The “r” controls the vowel
sound, causing the a to have a unique sound.
3. The correct answer is (A). Fluency refers to reading
smoothly, quickly, and with expression. (A) offers the
nonfluent student opportunities to engage in
meaningful literary experiences while gaining courage
and self esteem, while also experiencing ownership.
4. The correct answer is (B). The reader can follow the
link provided to easily seek more information
5. The correct answer is (D). The images in a graphic
novel provide information not included within the text,
such as character attributes. In a graphic novel, the
combination of text and images is required to produce
the complete story.
6. The correct answer is (C). The qualitative attributes
are subjective and can only be evaluated by a human
reader (i.e., “predictability of text”).
7. The correct answers are (B) and (D). A technical
speech and a research report both require that
information be collected and synthesized.
8. The correct answer is (A). Blogging programs
facilitate personal writing and typically have open
forums that encourage readers to respond to the
writing with written discussion.
9. The correct answer is (B). A metaphor is a type of
figurative language in which one image or idea is
connected with another. In literature, the word dust is
often associated with death because life forms decay
into soil after death.
10. The correct answer is (C). The word transitory
refers to change, and the speaker mentions a desire to
reject things that turn to dust, acquire dust, and start to
fade. These are all types of change.
11. The correct answer is (B). This sentence has two
independent clauses joined by a semicolon, including
one independent clause with a compound subject.
12. The correct answer is (B). The conclusion of an oral
presentation usually contains a clear summary of the
main points to reinforce the presentations goal.
13. The correct answer is (B). The statement
demonstrates paraphrasing of others statements. An
active listener spends more time listening than talking,
which paraphrasing demonstrates.
14. The correct answer is (D). The question requires an
understanding of the properties of operations. An
operation is commutative on a set of numbers if the
operation can be performed on any two numbers from
the set and produce the same result regardless of the
order in which the numbers are written. The operation
of addition is commutative on the set of real numbers,
since for any two real numbers a and b, a + b = b + a,
i.e., the sum is the same regardless of the order in which
the numbers are added. The commutative property of
addition thus guarantees that 8 + 9 = 9 + 8.
15. The correct answer is (C). The question requires an
understanding of how to identify relationships between
the corresponding terms of two numerical patterns. The
rule that relates x and y can be found by finding the
rate of change using the table. For every 3-unit increase
in x , y decreases by 18 units, that is, for each 1-unit
increase in x, y decreases by
18
3
6= units. Therefore,
the rate of change is 6, and the rule that relates x and
y is the equation in (C).
16. The correct answer is (C). The question requires an
understanding of how to solve problems involving the
measurement of elapsed time. Between 1:30 p.m. and
3:10 p.m. there are 1 hour and 40 minutes, or 100
minutes.
17. The correct answer is (A). The question requires an
understanding of how to interpret probabilities relative
to likelihood of occurrence. The coin has 2 possible
outcomes: heads or tails. The probability of the coin
landing heads up is 1 out of 2, or 1
2
. The cube has 6
possible outcomes: 1, 2, 3, 4, 5, or 6. The probability of
the face numbered 4 landing up is 1 out of 6, or 1
6
. To
find the combined probability, multiply the probabilities
of the two independent events together, i.e.,
1
2
1
6
1
12
×= .
Answers to Sample Questions
The Praxis® Study Companion 27
Step 3: Practice with Sample Test Questions
18. The correct answer is (A). The question requires an
understanding of how to find factors and multiples of
numbers. The prime factorization of 18 is
23
2
×
and
the prime factorization of 28 is
27
2
×
. So the prime
factorization of
18 28×
is
237
32
××
., which consists
only of the prime factors 2, 3, and 7.
19. The correct answer is (A). The question requires an
understanding of how to solve multistep one-variable
linear inequalities. The addition property of inequalities
states that for any real numbers a, b, and c, if
ab
,
then
acbc+≤+
and if
ab
, then
acbc+≥+
.
Adding −5 to both sides of the inequality yields the
equivalent inequality
02≤−
p. Adding
p
to both sides
of the new inequality yields the equivalent inequality
p
2
.
20. The correct answer is (D). The question requires an
understanding of how to use attributes to classify or
draw polygons. A polygon with six sides is called a
hexagon. A polygon for which any two points on the
polygon can be connected by a segment and the
segment is entirely contained within the interior of the
polygon is a convex polygon; a polygon for which this
is not true is a concave polygon. A regular polygon is a
polygon whose sides are all congruent and whose
angles all have the same measure. The polygon shown
is not regular but is a concave hexagon.
21. The correct answer is (C). The question requires an
understanding of how to use rounding strategies to
solve problems and determine the reasonableness of
results. The average number of sandwiches served in
each lunch period is found by calculating
157 4÷
. To
estimate this value, 157 can be rounded to 160, the
number closest to 157 that is a multiple of 4. The
estimate can be produced by mentally finding the
value of
160 4÷
, i.e., 40.
22. The correct answer is (B). The question requires an
understanding of how to translate between verbal
statements and algebraic expressions or equations.
Jack made 12 dollars per hour at his first job. He made 4
dollars per half hour, or
428×=
dollars per hour, at his
second job. Finally, Jack made 5 dollars for each 20
minutes, or
5315×=
dollars per hour, at his third job. If
he worked
H
hours at each job, he made
12 815×+×+ ×HH H
.
23. The correct answer is (D). The question requires an
understanding of how to use proportional relationships
to solve ratio problems. With 1 cup of blueberry juice
and 3 cups of red raspberry juice, Edie can make 4 cups
of punch. In the juice mixture, 1
4
of the total number of
cups is blueberry and 3
4
of the total number of cups is
red raspberry. Thus, in 48 cups of juice mixture, the
number of cups of red raspberry juice needed is 3
4
of
48, which equals 36 cups.
24. The correct answer is (A). Mount Rainier is located
in the state of Washington. The greatest single-peak
glacial system in the United States radiates from this
dormant volcano in the Cascade Mountains.
25. The correct answer is (C). During the Ice Age, the
level of the water in the Pacific Ocean lowered,
exposing a land bridge across the Bering Strait. The cold
northern climate encouraged many people to migrate
throughout the continent in search of better living
conditions.
26. The correct answer is (C). The creation of a third
party in national politics would be a political action, not
one of civil rights.
27. The correct answer is (D). Article 1 Section 2 of the
Constitution of the United States says, The House of
Representatives shall be composed of Members chosen
every second Year by the People… . All members of the
House are elected at the same time every two years.
28. The correct answer is (B). In the 15th century AD,
explorers from Portugal encountered the social system
of India and called these groups castes. As time went
on, the four basic castes gradually grew more complex,
with hundreds of subdivisions.
29. The correct answer is (B). Of the nine countries
shown in the graph, the Soviet Union and Iraq were the
two countries that produced more oil in 1975 than
in 1974.
30. The correct answer is (A). Volcanic activity is the
only process by which material from inside Earth is
brought to the surface. The other processes are means
of wearing down Earth’s surface.
31. The correct answer is (B). At the first lunar quarter
the Sun, Earth, and Moon form a right triangle, with
Earth at the right angle, so that the half of the Moon
facing Earth appears half lighted and half dark.
The Praxis® Study Companion 28
Step 3: Practice with Sample Test Questions
32. The correct answer is (B). Perspiring is an
adaptation that allows mammals to lose heat. When
the body temperature rises, sweat is produced. As the
water in the sweat evaporates, the skin is cooled, not
warmed.
33. The correct answer is (D). In a vacuum, the only
external force acting on each of the objects would be
the gravitational force of Earth. This gravitational force
is equal to M × g, where M is the objects mass and g is
the constant acceleration of gravity (9.8 meters per
second squared). According to Newtons second law,
the acceleration, a, of an object times its mass is equal
to the external force acting on it. For this situation,
Newtons second law gives M × a = M × g , or a = g.
Thus, in a vacuum all objects fall freely with the same
constant acceleration g, regardless of their mass.
34. The correct answer is (A). To find the volume of a
large rectangular block of wood, first use the metric
ruler to find the length, width, and height of the block.
Then use the formula for the volume of a rectangular
solid—length × width × height— to determine the
volume.
35. The correct answer is (C). A hypothesis is a best
guess or a possible explanation of a scientific problem.
Scientific experimentation can either support or fail to
support the hypothesis.
36. The correct answer is (A). When putting living
things into a biological classification scheme, the
broadest category is kingdom, followed by phylum,
class, order, family, genus, and species.
The Praxis® Study Companion 29
Step 4: Determine Your Strategy for Success
4. Determine Your Strategy for Success
Set clear goals and deadlines so your test preparation is focused and ecient
Effective Praxis test preparation doesn’t just happen. You’ll want to set clear goals and deadlines for yourself
along the way. Otherwise, you may not feel ready and confident on test day.
1) Learn what the test covers.
You may have heard that there are several different versions of the same test. Its true. You may take one
version of the test and your friend may take a different version a few months later. Each test has different
questions covering the same subject area, but both versions of the test measure the same skills and
content knowledge.
You’ll find specific information on the test you’re taking on page 5, which outlines the content categories
that the test measures and what percentage of the test covers each topic. Visit www.ets.org/praxis/
testprep for information on other Praxis tests.
2) Assess how well you know the content.
Research shows that test takers tend to overestimate their preparedness—this is why some test takers
assume they did well and then find out they did not pass.
The Praxis tests are demanding enough to require serious review of likely content, and the longer you’ve
been away from the content, the more preparation you will most likely need. If it has been longer than a few
months since you’ve studied your content area, make a concerted effort to prepare.
3) Collect study materials.
Gathering and organizing your materials for review are critical steps in preparing for the Praxis tests. Consider
the following reference sources as you plan your study:
Did you take a course in which the content area was covered? If yes, do you still have your books or
your notes?
Does your local library have a high school-level textbook in this area? Does your college library have a
good introductory college-level textbook in this area?
Practice materials are available for purchase for many Praxis tests at www.ets.org/praxis/testprep. Test
preparation materials include sample questions and answers with explanations.
4) Plan and organize your time.
You can begin to plan and organize your time while you are still collecting materials. Allow yourself plenty of
review time to avoid cramming new material at the end. Here are a few tips:
Choose a test date far enough in the future to leave you plenty of preparation time. Test dates can be
found at http://www.ets.org/praxis/register/centers_dates/.
Work backward from that date to figure out how much time you will need for review.
Set a realistic schedule—and stick to it.
The Praxis® Study Companion 30
Step 4: Determine Your Strategy for Success
5) Practice explaining the key concepts.
Praxis tests with constructed-response questions assess your ability to explain material effectively. As a
teacher, you’ll need to be able to explain concepts and processes to students in a clear, understandable
way. What are the major concepts you will be required to teach? Can you explain them in your own words
accurately, completely, and clearly? Practice explaining these concepts to test your ability to effectively
explain what you know.
6) Understand how questions will be scored.
Scoring information can be found on page 58.
7) Develop a study plan.
A study plan provides a road map to prepare for the Praxis tests. It can help you understand what skills and
knowledge are covered on the test and where to focus your attention. Use the study plan template on page
34 to organize your efforts.
And most important—get started!
Would a Study Group Work for You?
Using this guide as part of a study group
People who have a lot of studying to do sometimes find it helpful to form a study group with others who are
working toward the same goal. Study groups give members opportunities to ask questions and get detailed
answers. In a group, some members usually have a better understanding of certain topics, while others in the
group may be better at other topics. As members take turns explaining concepts to one another, everyone
builds self-confidence.
If the group encounters a question that none of the members can answer well, the group can go to a teacher or
other expert and get answers efficiently. Because study groups schedule regular meetings, members study in a
more disciplined fashion. They also gain emotional support. The group should be large enough so that multiple
people can contribute different kinds of knowledge, but small enough so that it stays focused. Often, three to
six members is a good size.
Here are some ways to use this guide as part of a study group:
Plan the groups study program. Parts of the study plan template, beginning on page 34 can help
to structure your groups study program. By filling out the first five columns and sharing the worksheets,
everyone will learn more about your groups mix of abilities and about the resources, such as textbooks, that
members can share with the group. In the sixth column (“Dates I will study the content”), you can create an
overall schedule for your groups study program.
Plan individual group sessions. At the end of each session, the group should decide what specific
topics will be covered at the next meeting and who will present each topic. Use the topic headings and
subheadings in the Test at a Glance table on page 5 to select topics, and then select practice questions,
beginning on page 20.
Prepare your presentation for the group. When it’s your to turn present, prepare something that is
more than a lecture. Write two or three original questions to pose to the group. Practicing writing actual
questions can help you better understand the topics covered on the test as well as the types of questions
you will encounter on the test. It will also give other members of the group extra practice at answering
questions.
The Praxis® Study Companion 31
Step 4: Determine Your Strategy for Success
Take a practice test together. The idea of a practice test is to simulate an actual administration of the
test, so scheduling a test session with the group will add to the realism and may also help boost everyones
confidence. Remember, complete the practice test using only the time that will be allotted for that test on
your administration day.
Learn from the results of the practice test. Review the results of the practice test, including the
number of questions answered correctly in each content category. For tests that contain constructed-
response questions, look at the Sample Test Questions section, which also contain sample responses to
those questions and shows how they were scored. Then try to follow the same guidelines that the test
scorers use.
Be as critical as you can. You’re not doing your study partner(s) any favors by letting them get away with
an answer that does not cover all parts of the question adequately.
Be specic. Write comments that are as detailed as the comments about the sample responses. Indicate
where and how your study partner(s) are doing an inadequate job of answering the question. Writing notes
in the margins of the answer sheet may also help.
Be supportive. Include comments that point out what your study partner(s) got right.
Then plan one or more study sessions based on aspects of the questions on which group members performed
poorly. For example, each group member might be responsible for rewriting one paragraph of a response in
which someone else did an inadequate job.
Whether you decide to study alone or with a group, remember that the best way to prepare is to have an
organized plan. The plan should set goals based on specific topics and skills that you need to learn, and it
should commit you to a realistic set of deadlines for meeting those goals. Then you need to discipline yourself
to stick with your plan and accomplish your goals on schedule.
The Praxis® Study Companion 32
Step 5: Develop Your Study Plan
5. Develop Your Study Plan
Develop a personalized study plan and schedule
Planning your study time is important because it will help ensure that you review all content areas covered on the
test. Use the sample study plan below as a guide. It shows a plan for the Core Academic Skills for Educators: Reading
test. Following that is a study plan template that you can fill out to create your own plan. Use the “Learn about Your
Test and Test Specifications" information beginning on page 5 to help complete it.
Use this worksheet to:
1. Dene Content Areas: List the most important content areas for your test as defined in chapter 1.
2. Determine Strengths and Weaknesses: Identify your strengths and weaknesses in each content area.
3. Identify Resources: Identify the books, courses, and other resources you plan to use for each content area.
4. Study: Create and commit to a schedule that provides for regular study periods.
Praxis Test Name (Test Code): Core Academic Skills for Educators: Reading (5712)
Test Date: 9/15/15
Content covered Description
of content
How well do
I know the
content?
(scale 1–5)
What
resources do I
have/need for
the content?
Where can I
nd the
resources I
need?
Dates I will
study the
content
Date
completed
Key Ideas and Details
Close reading
Draw inferences and
implications from the
directly stated content
of a reading selection
3
Middle school
English
textbook
College library,
middle school
teacher
7/15/15 7/15/15
Determining Ideas
Identify summaries or
paraphrases of the main
idea or primary purpose
of a reading selection
3
Middle school
English
textbook
College library,
middle school
teacher
7/17/15 7/17/15
Determining Ideas
Identify summaries
or paraphrases of the
supporting ideas and
specific details in a
reading selection
3
Middle and
high school
English
textbook
College library,
middle and
high school
teachers
7/20/15 7/21/15
Craft, Structure, and Language Skills
Interpreting tone
Determine the authors
attitude toward material
discussed in a reading
selection
4
Middle and
high school
English
textbook
College library,
middle and
high school
teachers
7/25/15 7/26/15
Analysis of
structure
Identify key transition
words and phrases in a
reading selection and
how they are used
3
Middle and
high school
English
textbook,
dictionary
College library,
middle and
high school
teachers
7/25/15 7/27/15
Analysis of
structure
Identify how a reading
selection is organized
in terms of cause/effect,
compare/contrast,
problem/solution, etc.
5
High school
textbook,
college course
notes
College library,
course notes,
high school
teacher, college
professor
8/1/15 8/1/15
Authors purpose
Determine the role that
an idea, reference, or
piece of information
plays in an authors
discussion or argument
5
High school
textbook,
college course
notes
College library,
course notes,
high school
teacher, college
professor
8/1/15 8/1/15
(continued on next page)
The Praxis® Study Companion 33
Step 5: Develop Your Study Plan
Content covered Description
of content
How well do
I know the
content?
(scale 1–5)
What
resources do I
have/need for
the content?
Where can I
nd the
resources I
need?
Dates
I will
study the
content
Date
completed
Language in
different contexts
Determine whether
information presented
in a reading selection
is presented as fact or
opinion
4
High school
textbook,
college course
notes
College library,
course notes,
high school
teacher, college
professor
8/1/15 8/1/15
Contextual
meaning
Identify the meanings of
words as they are used in
the context of a reading
selection
2
High school
textbook,
college course
notes
College library,
course notes,
high school
teacher, college
professor
8/1/15 8/1/15
Figurative
Language
Understand figurative
language and nuances in
word meanings
2
High school
textbook,
college course
notes
College library,
course notes,
high school
teacher, college
professor
8/8/15 8/8/15
Vocabulary range
Understand a range
of words and phrases
sufficient for reading at
the college and career
readiness level
2
High school
textbook,
college course
notes
College library,
course notes,
high school
teacher, college
professor
8/15/15 8/17/15
Integration of Knowledge and Ideas
Diverse media and
formats
Analyze content
presented in diverse
media and formats,
including visually and
quantitatively, as well as
in words
2
High school
textbook,
college course
notes
College library,
course notes,
high school
teacher, college
professor
8/22/15 8/24/15
Evaluation of
arguments
Identify the relationship
among ideas presented
in a reading selection
4
High school
textbook,
college course
notes
College library,
course notes,
high school
teacher, college
professor
8/24/15 8/24/15
Evaluation of
arguments
Determine whether
evidence strengthens,
weakens, or is relevant
to the arguments in a
reading selection
3
High school
textbook,
college course
notes
College library,
course notes,
high school
teacher, college
professor
8/27/15 8/27/15
Evaluation of
arguments
Determine the logical
assumptions upon
which an argument or
conclusion is based
5
High school
textbook,
college course
notes
College library,
course notes,
high school
teacher, college
professor
8/28/15 8/30/15
Evaluation of
arguments
Draw conclusions from
material presented in a
reading selection
5
High school
textbook,
college course
notes
College library,
course notes,
high school
teacher, college
professor
8/30/15 8/31/15
Comparison of
texts
Recognize or predict
ideas or situations that
are extensions of or
similar to what has been
presented in a reading
selection
4
High school
textbook,
college course
notes
College library,
course notes,
high school
teacher, college
professor
9/3/15 9/4/15
Comparison of
texts
Apply ideas presented
in a reading selection to
other situations
2
High school
textbook,
college course
notes
College library,
course notes,
high school
teacher, college
professor
9/5/15 9/6/15
The Praxis® Study Companion 34
Step 5: Develop Your Study Plan
My Study Plan
Use this worksheet to:
1. Dene Content Areas: List the most important content areas for your test as defined in chapter 1.
2. Determine Strengths and Weaknesses: Identify your strengths and weaknesses in each content area.
3. Identify Resources: Identify the books, courses, and other resources you plan to use for each content area.
4. Study: Create and commit to a schedule that provides for regular study periods.
Praxis Test Name (Test Code): ____________________________________________________________
Test Date: _____________
Content covered Description
of content
How well do
I know the
content?
(scale 1–5)
What
resources do I
have/need for
this content?
Where can I
nd the
resources I
need?
Dates I will
study this
content
Date
completed
(continued on next page)
The Praxis® Study Companion 35
Step 5: Develop Your Study Plan
Content covered Description
of content
How well do
I know the
content?
(scale 1–5)
What
resources do I
have/need for
the content?
Where can I
nd the
resources I
need?
Dates I will
study the
content
Date
completed
The Praxis® Study Companion 36
Step 6: Review Study Topics
6. Review Study Topics
Detailed study topics with questions for discussion
Using the Study Topics That Follow
The Elementary Education: Content Knowledge test is designed to measure the knowledge and skills necessary
for a beginning teacher.
This chapter is intended to help you organize your preparation for the test and to give you a clear indication of
the depth and breadth of the knowledge required for success on the test.
Virtually all accredited programs address the topics covered by the test; however, you are not expected to be an
expert on all aspects of the topics that follow.
You are likely to find that the topics below are covered by most introductory textbooks. Consult materials and
resources, including lecture and laboratory notes, from all your coursework. You should be able to match up
specific topics and subtopics with what you have covered in your courses.
Try not to be overwhelmed by the volume and scope of content knowledge in this guide. Although a specific
term may not seem familiar as you see it here, you might find you can understand it when applied to a real-life
situation. Many of the items on the actual test will provide you with a context to apply to these topics or terms.
Discussion Areas
Interspersed throughout the study topics are discussion areas, presented as open-ended questions or
statements. These discussion areas are intended to help test your knowledge of fundamental concepts and your
ability to apply those concepts to situations in the classroom or the real world. Most of the areas require you
to combine several pieces of knowledge to formulate an integrated understanding and response. If you spend
time on these areas, you will gain increased understanding and facility with the subject matter covered on the
test. You may want to discuss these areas and your answers with a teacher or mentor.
Note that this study companion does not provide answers for the discussion area questions, but thinking about the
answers to them will help improve your understanding of fundamental concepts and will probably help you
answer a broad range of questions on the test.
The Praxis® Study Companion 37
Step 6: Review Study Topics
Study Topics
An overview of the areas covered on the test, along
with their subareas, follows.
I. Reading and Language Arts
A. Reading: Foundational Skills
1. Understands the role of phonological
awareness in literacy development
a. Explains the importance of phonological
awareness as a foundational skill for literacy
development
b. Identifies and provides examples of
phonemes, syllables, onsets, and rimes
c. Identifies and provides examples of
blending, segmenting, substituting, and
deleting phonemes, syllables, onsets, rimes
2. Understands the role of phonics and word
analysis in literacy development
a. Explains the importance of phonics and
word analysis in literacy development
b. Distinguishes among common letter-sound
correspondences and spelling conventions
c. Distinguishes high-frequency sight words
from decodable words appropriate for
particular grades
d. Identifies roots and affixes to decode
unfamiliar words
e. Recognizes various stages of language
acquisition (e.g., WIDA taxonomy)
f. Delineates common phonics and word-
recognition approaches for ELLs (pedagogy)
g. Differentiates syllabication patterns (e.g.,
open, closed, CVe)
3. Understands the role of fluency (e.g., rate,
accuracy) in literacy development
a. Defines fluency and related terms (e.g.,
accuracy, rate, prosody)
b. Explains the impact of fluency on
comprehension
B. Reading: Literature and Informational Text
1. Understands how to use key ideas and details
to comprehend literature and informational
text
a. Identifies the key details, moral, and/or
theme of a literary text, citing specific
textual evidence
b. Identifies the key details and/or central idea
of an informational text, citing specific
textual evidence
c. Makes inferences from a text and supports
them with appropriate evidence
d. Summarizes information from a text
e. Analyzes the characters, setting, and plot of
a literary text
f. Analyzes the relationships among
individuals, events, ideas, and concepts in
an informational text
2. Understands how features and structures of
text across genres affect comprehension
a. Identifies structural elements of literature
across genres (e.g., casts of characters and
stage directions in drama, rhyme and meter
in poetry)
b. Uses text features (e.g., headings, sidebars,
hyperlinks) to locate information in a print
or digital informational text
c. Identifies organizational structures of
informational text (e.g., cause/effect,
problem/solution)
d. Identifies how structural elements
contribute to the development of a literary
text as a whole
3. Understands the concept of point of view
using evidence from the text
a. Identifies authors point of view in various
genres and supports conclusions with
evidence from the text
b. Compares multiple accounts of the same
event or topic to identify similarities or
differences in point of view
c. Identifies how point of view impacts the
overall structure of a literary or
informational text
The Praxis® Study Companion 38
Step 6: Review Study Topics
4. Understands how to integrate and compare
written, visual, and oral information from texts
and multimedia sources
a. Explains how visual and oral elements
enhance the meaning and effect of a
literary text (e.g., picture book, graphic
novel, multimedia presentation of a folktale)
b. Compares the written version of a literary
text with an oral, staged, or filmed version
c. Compares two or more literary texts that
address the same theme
d. Compares two or more informational texts
that address the same topic
e. Interprets visual and multimedia elements
in literary and informational texts
f. Evaluates key claims in a text and supports
them with reasons and evidence from the
text
5. Knows the role of text complexity in reading
development
a. Explains the three factors (i.e., quantitative,
qualitative, and reader and task) that
measure text complexity
b. Identifies features of text-leveling systems
C. Writing
1. Understands the characteristics of common
types of writing
a. Distinguishes among common types of
writing (e.g., opinion/argument,
informative/explanatory, narrative)
b. Identifies the purpose, key components,
and subgenres (e.g., speeches,
advertisements, narrative poems) of each
common type of writing
c. Evaluates the effectiveness of writing
samples of each type
2. Understands the characteristics of effective
writing
a. Evaluates the appropriateness of a
particular piece of writing for a specific task,
purpose, and audience
b. Evaluates the development, organization, or
style of a piece of writing
c. Identifies appropriate revisions to
strengthen a piece of writing
d. Writes clearly and coherently
e. Identifies the interrelationships among
planning, revising, and editing in the
process of writing
3. Knows the developmental stages of writing
(e.g., picture, scribble)
a. Identifies the grade-appropriate continuum
of student writing
4. Knows the importance of digital tools for
producing and publishing writing and for
interacting with others
a. Identifies the characteristics and purposes
of a variety of digital tools for producing
and publishing writing
b. Identifies the purposes of a variety of digital
tools for interacting with others
5. Knows the research process
a. Identifies the steps in the research process
b. Distinguishes between primary and
secondary sources and their uses
c. Distinguishes between reliable and
unreliable sources
d. Distinguishes between paraphrasing and
plagiarizing
e. Knows how to locate credible print and
digital sources, locate information within
the sources, and cite the sources
D. Language
1. Knows the conventions of standard English
grammar, usage, mechanics, and spelling when
writing, speaking, reading, and listening
a. Explains the function of different parts of
speech
b. Corrects errors in usage, mechanics, and
spelling
c. Identifies examples of different sentence
types (e.g., simple, compound, compound-
complex)
d. Identify how varieties of English (e.g.,
dialects, registers) used in stories, dramas, or
poems support the overall meaning
2. Understands how to determine the meaning of
words and phrases
a. Determines the literal meaning of unknown
words and phrases from context, syntax,
and/or knowledge of roots and affixes
b. Identifies types of figurative language
The Praxis® Study Companion 39
Step 6: Review Study Topics
c. Interprets figurative language
d. Analyzes the relationship between word
choice and tone in a text
3. Understands characteristics of conversational,
academic, and domain-specific language
a. Differentiates among the three tiers of
vocabulary
b. Identifies relevant features of language such
as word choice, order, and punctuation
E. Speaking and Listening
1. Knows the characteristics of effective
collaboration to promote comprehension
a. Identifies techniques to communicate for a
variety of purposes with diverse partners
b. Identifies the characteristics of active
listening
2. Knows the characteristics of engaging oral
presentations
a. Identifies elements of engaging oral
presentations (e.g., volume, articulation,
awareness of audience)
Discussion areas: English Language Arts
What is the difference between a consonant
digraph and a consonant blend?
What are the differences between the
syntactic, semantic, and graphophonic
cueing systems?
Compare the basic components of teaching
phonological awareness, such as blending,
segmenting, substitution, and deletion.
How does fluency impact comprehension?
What are the most effective methods for
teaching reading to English-language
learners?
What are the elements of reading fluency?
What is the difference between quantitative
and qualitative measures when evaluating
text complexity?
How can strategies like repeated reading and
choral reading improve students’ fluency?
What is the difference between prosody and
automaticity?
What are the best methods for vocabulary
acquisition?
What is the difference between the three
tiers of vocabulary?
How can students use roots and affixes,
syntax, and context clues to help determine
word meaning?
How can students use metacognition to aid
in reading comprehension?
What is the difference between literal and
inferential comprehension?
What are some effective methods to activate
students’ prior knowledge before reading?
What are the benefits of graphics, images,
and pictures for readers?
What are the common features of
informational texts, and how can they best
guide students?
What are the common types of
organizational structure of informational
texts and how do they differ?
What is the difference between factual
writing and opinion writing?
What are the elements of narrative texts?
What are the differences between some of
the subgenres of fiction such as fantasy,
science fiction, mystery, and folktales?
What are the best methods for teaching
poetic elements like mood, rhythm, and
structure?
Define and compare the different types of
figurative language, such as simile, metaphor,
hyperbole, personification, and
onomatopoeia, etc.
Which graphic organizers are most
appropriate for the different types of writing
and reading?
How can students ensure that online
resources are credible and unbiased?
What is the difference between a primary
and a secondary source?
List and define the various parts of speech.
What is pronoun-antecedent agreement?
What is subject-verb agreement?
The Praxis® Study Companion 40
Step 6: Review Study Topics
What are the rules for the different types of
sentences: simple, compound, complex, and
compound-complex?
What are the differences between
declarative, interrogative, imperative, and
exclamatory sentences?
What are the most effective ways to guide
students to find the appropriate tone,
purpose, and audience for the various types
of writing (narrative, persuasive, informative,
and descriptive)?
What are the different stages of writing
development, and what can a student do at
each stage?
What is the difference between the revision
stage and the editing stage of the writing
process?
Which are the most appropriate methods of
formative and summative assessments in
writing?
What are some appropriate and effective
ways to utilize technology in the classroom,
especially as part of the writing process?
What elements, both verbal and nonverbal,
make for an effective oral presentation?
How can a teacher both ensure and assess
active listening in students?
How can a student best use multimedia in a
presentation?
What are the most effective ways to
encourage collaboration in the classroom?
How can different types of discussion
groups, e.g., fishbowl, jigsaw, and literature
circles, be utilized to encourage collaboration
and comprehension?
II. Mathematics
A. Numbers and Operations
1. Understands the place value system
a. Writes numbers using base-10 numerals,
number names, and expanded form
b. Composes and decomposes multi-digit
numbers
c. Given a digit, identifies the place the digit is
in and its value in that place
d. Recognizes that a digit in one place
represents ten times what it represents in
the place to its right and one-tenth what it
represents in the place to its left, and
extends this recognition to several places to
the right or left
e. Uses whole-number exponents to denote
powers of 10
f. Rounds multi-digit numbers to any place
value
2. Understands operations and properties of
rational numbers
a. Solves multistep mathematical and real-
world problems using addition, subtraction,
multiplication, and division of rational
numbers and shows knowledge of how to
classify problem situations, inverse
operations, remainders, concepts of zero,
absolute value, and opposites
b. Understands various strategies and
algorithms used to perform operations on
rational numbers
c. Recognizes concepts of rational numbers
and their operations, including those
related to unit fractions, composition and
decomposition of fractions, comparing
fractions
d. Solves problems using the order of
operations, including problems involving
whole-number exponents
e. Identifies properties of operations (e.g.,
commutative, associative, distributive) and
uses them to solve problems
f. Represents rational numbers and their
operations in different ways, using
drawings, models, number lines, arrays
g. Compares, classifies, and orders rational
numbers
h. Converts between fractions, decimals, and
percents
The Praxis® Study Companion 41
Step 6: Review Study Topics
3. Understands proportional relationships and
percents
a. Applies the concepts of ratios and unit rates
to describe relationships between two
quantities
b. Understands percent as a rate per 100
c. Solves unit-rate problems
d. Uses proportional relationships to solve
ratio and percent problems
4. Knows how to use basic concepts of number
theory
a. Identifies and uses prime and composite
numbers
b. Finds factors and multiples of numbers
5. Knows a variety of strategies to determine
reasonableness of results
a. Recognizes the reasonableness of results
within the context of a given problem
b. Uses mental math, estimation, and
rounding strategies to solve problems and
determine reasonableness of results
B. Algebraic Thinking
1. Knows how to evaluate and manipulate
algebraic expressions, equations, and formulas
a. Differentiates between algebraic
expressions and equations
b. Adds and subtracts linear algebraic
expressions
c. Uses the distributive property to generate
equivalent linear algebraic expressions
d. Evaluates simple algebraic expressions (i.e.,
one variable, binomial) for given values of
variables
e. Uses mathematical terms to identify parts of
expressions and describe expressions
f. Translates between verbal statements and
algebraic expressions or equations (e.g., the
phrase “the number of cookies Joe has is
equal to twice the number of cookies Sue
has” can be represented by the equation
j = 2s )
g. Uses formulas to determine unknown
quantities
h. Differentiates between dependent and
independent variables in formulas
2. Understands the meanings of the solutions to
linear equations and inequalities
a. Solves multistep one-variable linear
equations and inequalities
b. Interprets solutions of multistep one-
variable linear equations and inequalities
(e.g., graphs the solution on a number line,
states constraints on a situation)
c. Uses linear relationships represented by
equations, tables, and graphs to solve
problems
3. Knows how to recognize and represent
patterns (e.g., number, shape)
a. Identifies, extends, describes, or generates
number and shape patterns
b. Makes conjectures, predictions, or
generalizations based on patterns
c. Identifies relationships between the
corresponding terms of two numerical
patterns (e.g., find a rule for a function
table)
C. Geometry and Measurement
1. Understands how to classify one-, two-, and
three-dimensional figures
a. Uses definitions to identify lines, rays, line
segments, parallel lines, and perpendicular
lines
b. Classifies angles based on their measure
c. Composes and decomposes two- and
three-dimensional shapes
d. Uses attributes to classify or draw polygons
and solids
2. Knows how to solve problems involving
perimeter, area, surface area, and volume
a. Represents three-dimensional figures with
nets
b. Use nets that are made of rectangles and
triangles to determine the surface area of
three-dimensional figures
c. Finds the area and perimeter of polygons,
including those with fractional side lengths
d. Finds the volume and surface area of right
rectangular prisms, including those with
fractional edge lengths
e. Determines how changes to dimensions
change area and volume
The Praxis® Study Companion 42
Step 6: Review Study Topics
3. Knows the components of the coordinate
plane and how to graph ordered pairs on the
plane
a. Identifies the x-axis, the y-axis, the origin,
and the four quadrants in the coordinate
plane
b. Solves problems by plotting points and
drawing polygons in the coordinate plane
4. Knows how to solve problems involving
measurement
a. Solves problems involving elapsed time,
money, length, volume, and mass
b. Measures and compares lengths of objects
using standard tools
c. Knows relative sizes of United States
customary units and metric units
d. Converts units within both the United
States customary system and the metric
system
D. Data, Statistics, and Probability
1. Is familiar with basic statistical concepts
a. Identifies statistical questions
b. Solves problems involving measures of
center (mean, median, mode) and range
c. Recognizes which measure of center best
describes a set of data
d. Determines how changes in data affect
measures of center or range
e. Describes a set of data (e.g., overall patterns,
outliers)
2. Knows how to represent and interpret data
presented in various forms
a. Interprets various displays of data (e.g., box
plots, histograms, scatterplots)
b. Identifies, constructs, and completes graphs
that correctly represent given data (e.g.,
circle graphs, bar graphs, line graphs,
histograms, scatterplots, double bar graphs,
double line graphs, box plots, and line
plots/dot plots)
c. Chooses appropriate graphs to display data
3. Is familiar with how to interpret the probability
of events
a. Interprets probabilities relative to likelihood
of occurrence
Discussion areas: Mathematics
Why is it that 3 is greater than 2, but 1
3 is less
than 1
2
?
Is the square of a number always greater
than the number? Consider numbers such as
3, −2, 1
4
, and 0.
Are 1 and 2 prime numbers? Why or why
not?
Is zero an even number or an odd number?
Is the sum of two even numbers always
even? What about the sum of two odd
numbers?
Make a factor tree for 60.
Why do we put an arrow on the end of a
number line?
Create two or three different ways of visually
representing the product of 2 and 4. Think of
objects that elementary students would
relate to.
If a movie ticket was $5 last week and this
week is $6, what was the percent increase?
If the scale used on a blueprint is 1 inch to 4
feet and the drawing of a room is 4.5 inches
wide, how wide is the room?
Is 60 kilograms a reasonable weight for a
6-year-old child? Explain by using a
benchmark for a kilogram (i.e., an easy-to-
manipulate translation to pounds).
Write a problem that uses the “working
backwards” method. Be sure to give the end
result from which to work.
How would you translate the following
statement into a mathematical expression
that includes variables? The number of red
chips is 3 more than the number of blue
chips.
In the previous example, if there are 41 blue
chips and red chips altogether, how many
are red chips?
What is the difference between an
expression and an equation?
The Praxis® Study Companion 43
Step 6: Review Study Topics
Why is 0
1
equal to 0, but 1
0 is not even
defined ? Consider using 20
5 = 4 and
relating it to 20 = 5 × 4 to explain this oddity.
Do rectangles that have the same perimeter
always have the same area?
For a given perimeter, what is the shape with
the greatest area?
If a figure is a rectangle, is it also a square?
If a figure is a square, is it also a rectangle?
What is the area of the following shape?
What are the volumes of the following
shapes?
Draw a trapezoid that can be subdivided into
four congruent right triangles.
Draw a trapezoid that can be subdivided into
three equilateral triangles.
Can a right triangle be isosceles?
What stays the same when a transformation
is applied?
What changes when a transformation is
applied?
Describe the transformation shown below.
What stays the same and what changes?
On a number line, draw arrows to model this
calculation: 16 − 8 + 4 − 2
On a grid, model 24 as a product of integers
in four different ways.
On a grid, model 24 as the product of mixed
numbers or decimals.
On a grid, model 24 as the product of
numbers that contain square roots.
1
2
1
4
1
8
+++ … is an infinite sequence.
Use a square to show that the sum of the
sequence is 1. Hint—start this way:
How could you estimate the surface area of
a soup can using a net of the can made from
graph paper?
What natural phenomena are the basis for
many of our time measurements?
Name some countries where the English
system is used and some countries where
the metric system is used.
Can a circle graph and a line graph display
the same information? Why or why not?
The Praxis® Study Companion 44
Step 6: Review Study Topics
How is a stem-and-leaf plot like a bar graph?
How is it different?
Describe a real-life use of a mode.
Describe a real-life situation that illustrates a
direct relationship.
Describe a real-life situation that illustrates an
inverse relationship.
Make a sample space for the possible
outcomes of the toss of three fair coins.
Explain why the computation 2 × 2 × 2 gives
the number of points in the sample space.
Is the average of two different numbers ever
greater than one of them?
Can I find the average of 10 numbers if I
know the sum of them but not the numbers
themselves?
Can I find the median of 10 numbers if I
know the sum of them but not the numbers
themselves?
III. Social Studies
A. Geography, Anthropology, and Sociology
1. Knows world and regional geography
a. Is familiar with spatial terms and can identify
spatial patterns of people, places, and
environments
b. Identifies the characteristics of places and
regions
c. Locates major physical features of
geography (e.g., mountain ranges, bodies of
water)
d. Locates major political features of
geography (e.g., continents, countries,
states, cities)
e. Demonstrates basic geographic literacy
(e.g., uses and interpretations of different
types of maps, understanding of the
concepts of absolute and relative location,
identification of cardinal and intermediate
directions)
2. Understands the interaction of physical and
human systems
a. Demonstrates knowledge of how humans
change the environment
b. Demonstrates knowledge of how the
environment affects human activities
c. Understands the importance of natural and
human resources
3. Knows the uses of geography
a. Applies geography to interpret the past
and the present and to plan for the future
b. Demonstrates knowledge of societys
groups, institutions, and organizations
c. Demonstrates knowledge of how human
behavior is influenced by society
B. World History
1. Knows the major contributions of classical
civilizations such as Egypt, Greece, and Rome
a. Demonstrates knowledge of how modern
civilizations reflect, mirror, and learn from
the contributions of ancient civilizations
2. Understands twentieth-century
developments and transformations in world
history
a. Demonstrates knowledge of the causes
and effects of the First and Second World
Wars and the Cold War
b. Demonstrates knowledge of technological
developments (e.g., transportation,
communication, tools)
c. Demonstrates knowledge of the causes
and effects of globalization
3. Understands the role of cross-cultural
comparisons in world history instruction
a. Demonstrates knowledge of various
psychological, sociological, and cultural
factors needed to assess the similarities
and/or diversities in two or more different
cultures or societies
C. United States History
1. Knows about the European exploration and
colonization of North America and growth
and expansion of the United States
a. Demonstrates knowledge of Native
American peoples and cultures
b. Demonstrates knowledge of the reasons for
the colonization of North America and the
development of the thirteen colonies
The Praxis® Study Companion 45
Step 6: Review Study Topics
c. Is familiar with the interactions between
Native American groups, colonists, and
European powers
2. Knows about the American Revolution and
the founding of the United States
a. Understands the causes and effects of the
American Revolution
b. Identifies key individuals and events during
the American Revolution
c. Demonstrates knowledge of the challenges
faced by the early republic (e.g., creation of
a democratic government)
3. Knows about the major events and
developments in United States history from
founding to present
a. Demonstrates knowledge of the causes and
effects of the territorial expansion of the
United States (e.g., concept of Manifest
Destiny; Louisiana Purchase; impact on
Native Americans; role of technological,
political, and economic developments)
b. Understands the causes and effects of the
Civil War (e.g., growth of sectionalism, the
abolition movement, the Underground
Railroad, the reasons for the succession of
the Confederate States, the role of Abraham
Lincoln, the purposes and challenges of
Reconstruction)
c. Demonstrates knowledge of the causes and
effects of industrialization, urbanization, and
immigration
d. Is familiar with major social and cultural
developments throughout United States
history
4. Knows about twentieth-century
developments and transformations in the
United States
a. Demonstrates knowledge of the causes and
effects of the Great Depression (e.g., New
Deal legislation)
b. Demonstrates knowledge of the causes and
effects of the First and Second World Wars
and the Cold War
c. Demonstrates knowledge of major
economic developments (e.g., assembly
line, mass production,) and the influence of
technological developments
5. Understands connections between the
causes and effects of events
a. Demonstrates the ability to draw
connections between the causes and
effects of significant events throughout
United States history
D. Government, Citizenship, and Democracy
1. Understands the nature, purpose, and forms
of government
a. Is familiar with the founding principles of
the United States government (e.g.,
republicanism, separation of powers,
checks and balances, popular sovereignty)
b. Demonstrates knowledge of federalism
(e.g., division of power between the
national and state governments)
c. Demonstrates knowledge of the powers of
the three branches of the federal
government and the interactions among
them
d. Is familiar with basic characteristics of
different political systems
2. Knows key documents and speeches in the
history of the United States
a. Is familiar with the purpose and contents of
the Declaration of Independence
b. Is familiar with the Articles of Confederation
c. Demonstrates knowledge of the structure
of government outlined in the United
States Constitution
d. Demonstrates knowledge of the rights and
protections guaranteed to United States
citizens by the Constitution
e. Is familiar with key documents and
speeches (e.g., Gettysburg Address)
3. Knows the rights and responsibilities of
citizenship in a democracy
a. Demonstrates knowledge of civic
participation (e.g., community service,
membership in civic organizations)
b. Demonstrates knowledge of the rights and
responsibilities of citizens in the United
States (e.g. voting, paying taxes, freedom of
speech)
E. Economics
1. Knows key terms and basic concepts of
economics
a. Demonstrates knowledge of supply and
demand
The Praxis® Study Companion 46
Step 6: Review Study Topics
b. Is familiar with concepts of scarcity, choice,
and opportunity cost
c. Demonstrates knowledge of the role of
money and resources in economic decision
making
2. Understands how economics affects
population, resources, and technology
a. Demonstrates an understanding of how
people use resources to generate wealth
and enhance their lives
b. Demonstrates an understanding of how
economics drives and is driven by
technological innovations
3. Understands the government’s role in
economics and the impact of economics on
government
a. Demonstrates knowledge of the federal
government’s role in regulating the
economy
b. Demonstrates knowledge of taxing and
spending
F. Social Studies as Inquiry and Social Studies
Processes
1. Understands social studies as inquiry
a. Demonstrates knowledge of questioning,
gathering data, and drawing reasonable
conclusions
2. Understands how to use resource and
research material in social studies
a. Understands how to evaluate the
appropriate uses of a variety of resources
b. Identifies primary and secondary sources
and demonstrates knowledge of the uses of
each
c. Demonstrates knowledge of fact and
opinion and knows the uses of each in
social studies
3. Understands process skills in social studies
a. Understands how to interpret different
types of information
b. Evaluates relationships among different
variables
c. Demonstrates ability to draw conclusions
using tools of the field
Discussion areas: Social Studies
What is “map projection and what kinds of
decisions does it force mapmakers to make?
What is the primary categorization of each of
these regions, and why? Arab world, North
Africa, Sub-Saharan Africa, Latin America, the
Caribbean, North America, Western Europe,
Eastern Europe, East Asia, South Central Asia,
Southeast Asia, and Oceania
What is the difference between weather and
climate?
How do earthquakes create mountain
ranges?
What kinds of physical systems led to the
creation of the Grand Canyon? What about
Yosemite Valley?
What kind of immigration patterns and
effects were created by the Great Irish
Famine?
What are the major effects on the
environment and people when radioactive
materials get into the environment either by
leakage from storage or by an accident?
List as many ways as you can that the
pyramids and burial customs of Egypt
reflected aspects of Egyptian political, social,
cultural, religious, bureaucratic (record
keeping and writing), and artistic systems,
elements, and values.
How were the concepts of citizenship and
democracy in ancient Greece similar and
different from contemporary United States
concepts of citizenship and democracy?
How does a comparison of life in Athens and
Sparta illuminate differences among nations
in the world today?
Important contributions (in drama, sculpture,
sports, architecture, mathematics, and
science) and the emphasis on human
achievement
How big did the Roman Empire get, with
what borders, at its largest? In comparison,
how small was it when it fell? What were the
main reasons for the success at its largest
point and its gradual shrinking?
The Praxis® Study Companion 47
Step 6: Review Study Topics
What has been the role of Islam in African
history?
Does the caste system survive in India today?
How has the caste system shaped Indias
social, cultural, economic, and political
histories?
What were the effects of Japans isolation
until the 1850s? How did Japan change after
Admiral Perrys opening” of Japan?
Why were the Spanish able to defeat the
Aztec and Incan empires?
What does “Renaissance” mean? Why was the
name given to this historical period?
What does the term “Renaissance man or
“Renaissance woman mean, and how is the
definition of the term (then and now) related
to what happened during the Renaissance
period?
How did the Scientific Revolution change the
way humans perceived themselves and the
universe and how did it change the methods
of human inquiry?
What are the main reasons that a global
culture emerged in the twentieth century?
What are the consequences of this global
culture?
What were the weaknesses in the Articles of
Confederation that eventually led to its
replacement by the Constitution? Why were
the Articles written in this way in the first
place?
What was the Marbury v. Madison decision in
the Supreme Court and what did it establish?
What was “Manifest Destiny” and how did it
influence the expansion of United States
territory?
Make your own “immigration timeline of the
nineteenth century, noting the decades
during which immigrants from various
countries or regions came to the United
States in large numbers.
Post-Civil War immigration can be viewed in
terms of the “melting pot” analogy or in
terms of “pluralism or “multiculturalism.
What does this distinction mean, and why is
it important?
What was the Supreme Courts decision in
Brown v. Board of Education of Topeka? How
was the later decision in University of
California v. Bakke related to another
important educational issue in the twentieth
century?
Compare the intended and unintended
consequences of feudalism, communism,
monarchy, and liberal democracies in some
well-known cases: in trying to solve some
problems, what problems did the founders
create?
Compare the major features of a democratic
government with those of other forms of
government.
Why were the Magna Carta, Mayflower
Compact, and the Declaration of
Independence such milestone documents in
the political history of the world?
How does the electoral college work?
What is the line of authority if the President
and vice president are incapacitated? Who
are the next few in line?
Why is it claimed that the concept of
scarcity is the basis for the discipline of
economics?
What are some of the government
mechanisms that have been used in the
United States for redistributing wealth? What
are the dangers of too much government
redistribution versus the dangers of too
much wealth concentrated in a small
percentage of the population?
The Praxis® Study Companion 48
Step 6: Review Study Topics
IV. Science
A. Earth and Space Science
1. Understands basic physical and historical
geology
a. Identify Earths basic structure (e.g., mantle,
core, geographical features such as
mountains, magnetic field)
b. Identify and describe types and
characteristics of rocks and minerals
c. Recognize processes involved in erosion,
weathering, and deposition of Earths
surface materials
d. Recognize Earths internal processes
including impact of plate tectonic theory
(e.g., volcanoes, earthquakes)
e. Identify key aspects of the water cycle (e.g.,
evaporation, condensation, precipitation,
runoff )
f. Recognize important events in Earths
geologic history and the importance of the
rock record and fossils
2. Is familiar with the structure and processes of
Earths hydrosphere
a. Identify the geographic location of Earths
oceans and seas and the processes involved
with tides and waves
b. Identify characteristics of lakes, streams
rivers, polar ice, icebergs, glaciers, and
groundwater
c. Identify the basic characteristics of Earth’s
atmosphere
d. Recognize the basic concepts of weather
(e.g., clouds, precipitation, hurricanes)
e. Identify factors that affect climate and
seasons (e.g., climate zones, proximity to
mountains and oceans)
3. Is familiar with astronomy
a. Identify the major features of the solar
system, including the Sun, the planets,
moons, asteroids, and comets
b. Recognize the interactions of the Earth-
Moon-Sun system (e.g., phases of the Moon,
eclipses, seasons, tides
c. Recognize the major features of the
universe a (e.g., galaxies, stars, black holes)
B. Life Sciences
1. Understands the basic structure and function
of cells and levels of organization in living
things
a. Identify the structure and function of cell
organelles (e.g., nucleus, cell membrane)
b. Recognize basic cell processes such as cell
division and photosynthesis
c. Identify the levels of organization (cells,
tissues, organs, organ systems)
2. Understands basic genetics and evolution
a. Apply basic genetics (e.g., relationship
between genes and traits)
b. Recognize the basic structure and function
of DNA and relationship to heredity
c. Recognize common human genetic
disorders
d. Identify processes by which species change
over time, including natural selection,
mutation, evolution
3. Knows the hierarchical classification scheme
and the characteristics of the major groups of
organisms
a. Identify elements of classification schemes
(e.g., kingdom, genus, species)
b. Identify major characteristics of common
types of organisms (e.g. amphibians,
reptiles, mammals, plants)
4. Knows the major structures and functions of
plant organs and systems
a. Identify the basic structure and function of
leaves, roots, and stems
b. Recognize key aspects of asexual and
sexual reproduction, development, and
growth
c. Recognize the uptake and transport of
nutrients and water
5. Knows the basic anatomy and physiology of
animals, including human body systems
a. Identify examples of exchange with the
environment involving the respiratory,
excretory, and digestive systems
b. Recognize key aspects of internal transport
and exchange in terms of the circulatory
system
c. Recognize key aspects of support and
movement in terms of the skeletal and
muscular systems
The Praxis® Study Companion 49
Step 6: Review Study Topics
d. Identify key aspects of reproduction and
development
e. Recognize the function of immune systems
f. Identify the functions of immune systems,
nervous systems, and endocrine systems
g. Recognize the importance of homeostasis
6. Knows key aspects of ecology
a. Recognize key relationships between and
among species such as territoriality,
predator-prey, and parasitism
b. Recognize key aspects of ecosystems (e.g.,
biomes, energy levels, food webs, effect of
disturbances)
C. Physical Sciences
1. Knows the basic structure and properties of
matter
a. Identify basic properties of solids, liquids
and gases (e.g., structure, density,
conductivity, solubility)
b. Identify and distinguish between elements,
atoms, compounds, molecules, and
mixtures
c. Describe the atomic model, including
electrons, protons, neutrons, atomic
number and atomic mass
d. Is familiar with the periodic table of the
elements, its symbols and the information it
provides
2. Knows the basic relationships between
energy and matter
a. Recognize that energy and matter is
conserved in various situations
b. Recognize how various forms of kinetic and
potential energy can be transformed from
one form to another
c. Identify the differences between chemical
and physical properties/changes
d. Describe methods of heat transfer
(convection, radiation, conduction)
e. Describe how the states of matter undergo
phase changes and the energy changes
involved
3. Understands basic chemical reactions
a. Identify the difference between covalent
and ionic bonding
b. Interpret simple chemical formulas
c. Recognize that chemical reactions involve
energy changes
d. Identify chemical and physical properties of
acids and bases and the pH scale
e. Recognize common types of chemical
reactions such as neutralization, oxidation,
and combustion
4. Understands basic concepts in mechanics
a. Describe motion in terms of distance,
speed, velocity, and acceleration
b. Describe the effect of forces on objects
(e.g., collisions, pendulums, friction)
c. Recognize the effect of gravity and
distinguish between mass and weight
d. Recognize forces and physical properties
involving fluids that determine whether
objects will sink or float
5. Understands basic concepts in electricity,
magnetism, waves, and optics
a. Describe basic characteristics of magnets
(e.g., magnetic poles, attraction, repulsion)
b. Recognize electrostatic attraction and
repulsion
c. Describe electricity in terms of the flow of
electrons and identify voltage sources
(batteries and generators)
d. Describe the basic phenomena involving
light (reflection, rainbows, mirrors, prisms)
e. Describe basic characteristics of sound
(pitch, loudness, the Doppler effect)
D. Impact of Science and Technology on Society
1. Knows the impact of science and technology
on the environment and society
a. Recognize the impact of air and water
pollution, greenhouse gases
b. Recognize the impact of production and
disposal of consumer products
c. Recognize the benefits of conservation and
recycling
d. Identify renewable and nonrenewable
energy resources
e. Identify the pros and cons of power
generation based on various sources (e.g.,
fossil, nuclear, water, wind, solar, biomass,
geothermal)
The Praxis® Study Companion 50
Step 6: Review Study Topics
2. Is familiar with applications of science and
technology in daily life and public health
a. Identify applications of chemical and
physical principles related to common
consumer products (e.g., acid-base
properties of orange juice, applications of
physics in devices such as lenses)
b. Identify common agricultural practices (e.g.,
genetically modified crops, use of
herbicides and insecticides)
c. Recognize the role of nutrition, disease, and
medicine (e.g., food preservation, vitamins,
vaccines, viruses)
d. Recognize applications of medical
technologies (e.g., MRIs, X-rays, radiation
therapy)
E. Science as Inquiry and Science Processes
1. Understands the basic elements of scientific
inquiry and how they are used
a. Identify hypotheses, theories, models, and
laws, and their role in scientific inquiry
b. Explain the role of the elements of
experimental design, including
independent and dependent variables,
controls, sources of error, and drawing
conclusions
c. Recognize that scientific knowledge is
subject to change, consistent with
evidence, based on reproducible evidence
and includes unifying concepts and
processes (e.g., systems, models, constancy
and change, equilibrium, form and
function)
d. Recognize how key concepts developed
over time and identify the contribution of
key historical figures (e.g., Newtons laws,
Marie Curie’s work with radioactivity,
Mendel’s development of basic genetics)
2. Understands the common methods and tools
used to gather and present reliable data
a. Identify common units of measurement
(e.g., meter, gram, liter)
b. Explain the appropriate use of common
measurement tools (e.g., thermometers,
barometers, balances)
c. Organize and present data (e.g., graphs,
tables, charts, maps)
3. Knows how to interpret and draw conclusions
from data presented in tables, graphs, charts,
and maps
a. Identify patterns and significant points in
data
b. Draw conclusions and make predictions
based on presented data
c. Recognize relationships between variables
d. Recognize the effect of error on data and
conclusions
4. Understands procedures for safe and correct
use of laboratory materials and equipment
a. Recognize safe and appropriate methods to
prepare materials for classroom use
(activities and demonstrations)
b. Recognize when and how to use standard
equipment in the laboratory (e.g.,
microscopes, graduated cylinders)
c. Explain the use of standard safety
equipment (e.g., eyewash stations, safety
showers)
d. Identify appropriate student apparel and
behavior (e.g., goggles, clothing, no eating
in lab)
e. Recognize emergency procedures for
mishaps (e.g., fires, chemical spills, injuries)
and evacuation procedures
Discussion areas: Science
What is the inside of Earth like?
What is the difference between rocks and
minerals?
What substances are found in concrete?
What are fossils and how are they formed?
In which layer of the atmosphere is the
aurora borealis displayed? What is the cause
of this natural light show?
What is air pressure and how is it measured?
Why do monuments in Egypt last for
thousands of years, while similar monuments
in northern climates deteriorate very
quickly?
What is the “Ring of Fire”?
What causes a volcano to erupt?
What causes earthquakes?
What causes tides? What do low tide and
“high tide” mean?
The Praxis® Study Companion 51
Step 6: Review Study Topics
The greatest difference in water level
between a low tide and a high tide occurs
because of what alignment of the Moon,
Sun, and Earth?
How do storms form?
How do oceans affect climate?
Why do the planets circle the Sun?
How does a solar eclipse occur?
How are the inner planets of the solar system
different from the outer planets?
What causes the seasons on Earth? What is
the positional relationship of the Sun and
Earth at each season?
Why do the stars appear to move across the
sky each night while the pattern of stars stays
the same?
Why do different stars appear during
different seasons?
Why does the position of a planet as seen
from Earth change in relation to the
background of stars?
Why do stars twinkle while planets do not?
Are most cells flat? What do
electronmicroscope pictures show us about
cell shape?
Why are roots, stems, and leaves important
to plants?
How does the human circulatory system
work?
How does the human digestive system
work?
What are dominant and recessive traits?
How can two parents with brown eyes have
a child with blue eyes?
What are the steps in complete
metamorphosis? Incomplete
metamorphosis?
What is meant by “survival of the fittest”?
What makes a plant bend toward the light?
What is the scientific term associated with
this?
How does the human body maintain a
constant body temperature?
What are adaptations?
What happens if certain kinds of organisms,
such as edible plants, are introduced or
removed from a food chain?
How do food chains become food webs?
Does air take up space?
Sometimes when two chemicals are
combined, a chemical reaction takes place.
What are some of the signs of such a
chemical reaction?
What is an example of a change of state?
Where are the protons located in an atom?
How long does it take for a car traveling 30
miles per hour to go 3 miles?
When a person is driving a car that is moving
at the same speed as another car next to it,
why does the second car appear to be still?
What causes an object in motion to
accelerate or slow down?
What is the difference between weight and
mass?
Describe various ways in which an object
can have several forces acting on it and still
be at rest.
How is the energy of a rock sitting on the
top of a hill different from the energy of a
rock sitting at the bottom of the same hill?
Why does rubbing your hands together
make them warmer?
Describe how energy is transformed from
potential energy to kinetic energy as a
bicycle travels downhill.
How do visible light waves differ from sound
waves and water waves?
What is an example of how each of the
nonvisible waves is used in day-to-day life?
What about the properties of light makes a
red apple appear red?
Is light that interacts with a mirror reflected
or refracted?
Which types of lenses magnify and which
types produce an image reduced in size?
How do lenses help nearsighted and
farsighted people?
What are the basic components of a simple
electric circuit?
How does a compass work?
Some appliances can convert electrical
energy to heat energy, light energy, and
energy of motion. Give an example of each.
The Praxis® Study Companion 52
Step 6: Review Study Topics
Why does the sound that accompanies a
lightning strike come after the flash of light?
What are echoes, and what causes them?
How does physical fitness help a person?
In what ways can infection spread?
How do drugs, alcohol, and tobacco affect
the body?
How do prescription medicines differ from
over-the-counter medicines?
What, historically, has been the purpose of
sending humans into space?
Give an example of a complex scientific
endeavor that involves the work of hundreds
of people. Give an example of a relatively
straightforward scientific endeavor that
involves the work of a single scientist.
How are control variables and experimental
variables used in scientific investigations?
How do different questions require different
approaches and tools in the investigation
stage?
How do mathematics and technology assist
in different kinds of scientific inquiry?
What are some examples of measuring
instruments?
How does skepticism relate to scientific
inquiry?
What is the role of ethics in scientific inquiry?
Other than the inquiry process, what
methods have led to important new
scientific ideas or discoveries?
Why is each idea considered a crucial
conceptual scheme or procedural scheme,
and how does each idea cut across the
scientific disciplines?
The Praxis® Study Companion 53
Step 7: Review Smart Tips for Success
7. Review Smart Tips for Success
Follow test-taking tips developed by experts
Learn from the experts. Take advantage of the following answers to questions you may have and practical tips
to help you navigate the Praxis test and make the best use of your time.
Should I Guess?
Yes. Your score is based on the number of questions you answer correctly, with no penalty or subtraction for an
incorrect answer. When you don’t know the answer to a question, try to eliminate any obviously wrong answers
and then guess at the correct one. Try to pace yourself so that you have enough time to carefully consider
every question.
Can I answer the questions in any order?
You can answer the questions in order or skip questions and come back to them later. If you skip a question,
you can also mark it so that you can remember to return and answer it later. Remember that questions left
unanswered are treated the same as questions answered incorrectly, so it is to your advantage to answer every
question.
Are there trick questions on the test?
No. There are no hidden meanings or trick questions. All of the questions on the test ask about subject matter
knowledge in a straightforward manner.
Are there answer patterns on the test?
No. You might have heard this myth: the answers on tests follow patterns. Another myth is that there will never
be more than two questions in a row with the correct answer in the same position among the choices. Neither
myth is true. Select the answer you think is correct based on your knowledge of the subject.
Can I write on the scratch paper I am given?
Yes. You can work out problems on the scratch paper, make notes to yourself, or write anything at all. Your
scratch paper will be destroyed after you are finished with it, so use it in any way that is helpful to you. But make
sure to select or enter your answers on the computer.
Smart Tips for Taking the Test
1. Skip the questions you nd extremely dicult. Rather than trying to answer these on your first pass
through the test, you may want to leave them blank and mark them so that you can return to them later.
Pay attention to the time as you answer the rest of the questions on the test, and try to finish with 10 or
15 minutes remaining so that you can go back over the questions you left blank. Even if you don’t know
the answer the second time you read the questions, see if you can narrow down the possible answers, and
then guess. Your score is based on the number of right answers, so it is to your advantage to answer every
question.
The Praxis® Study Companion 54
Step 7: Review Smart Tips for Success
2. Keep track of the time. The on-screen clock will tell you how much time you have left. You will probably
have plenty of time to answer all of the questions, but if you find yourself becoming bogged down, you
might decide to move on and come back to any unanswered questions later.
3. Read all of the possible answers before selecting one. For questions that require you to select more
than one answer, or to make another kind of selection, consider the most likely answers given what the
question is asking. Then reread the question to be sure the answer(s) you have given really answer the
question. Remember, a question that contains a phrase such as Which of the following does NOT … is
asking for the one answer that is NOT a correct statement or conclusion.
4.
Check your answers. If you have extra time left over at the end of the test, look over each question and
make sure that you have answered it as you intended. Many test takers make careless mistakes that they
could have corrected if they had checked their answers.
5. Don’t worry about your score when you are taking the test. No one is expected to answer all of the
questions correctly. Your score on this test is not analogous to your score on the GRE® or other tests. It doesn’t
matter on the Praxis tests whether you score very high or barely pass. If you meet the minimum passing
scores for your state and you meet the states other requirements for obtaining a teaching license, you will
receive a license. In other words, what matters is meeting the minimum passing score. You can find passing
scores for all states that use The Praxis Series tests at
http://www.ets.org/s/praxis/pdf/passing_scores.pdf or on the Web site of the state for which you are
seeking certification/licensure.
6. Use your energy to take the test, not to get frustrated by it. Getting frustrated only increases stress
and decreases the likelihood that you will do your best. Highly qualified educators and test development
professionals, all with backgrounds in teaching, worked diligently to make the test a fair and valid measure
of your knowledge and skills. Your state painstakingly reviewed the test before adopting it as a licensure
requirement. The best thing to do is concentrate on answering the questions.
The Praxis® Study Companion 55
Step 8: Check on Testing Accommodations
8. Check on Testing Accommodations
See if you qualify for accommodations that may make it easier to take the Praxis test
What if English is not my primary language?
Praxis tests are given only in English. If your primary language is not English (PLNE), you may be eligible for
extended testing time. For more details, visit www.ets.org/praxis/register/accommodations/plne.
What if I have a disability or other health-related need?
The following accommodations are available for Praxis test takers who meet the Americans with Disabilities Act
(ADA) Amendments Act disability requirements:
Extended testing time
Additional rest breaks
Separate testing room
Writer/recorder of answers
Test reader
Sign language interpreter for spoken directions only
Perkins Brailler
Braille slate and stylus
Printed copy of spoken directions
Oral interpreter
Audio test
Braille test
Large print test book
Large print answer sheet
Listening section omitted
For more information on these accommodations, visit www.ets.org/praxis/register/disabilities.
Note: Test takers who have health-related needs requiring them to bring equipment, beverages, or snacks into
the testing room or to take extra or extended breaks must request these accommodations by following the
procedures described in the Bulletin Supplement for Test Takers with Disabilities or Health-Related Needs (PDF),
which can be found at http://www.ets.org/s/disabilities/pdf/bulletin_supplement_test_takers_with_
disabilities_health_needs.pdf.
You can find additional information on available resources for test takers with disabilities or health-related needs
at www.ets.org/disabilities.
The Praxis® Study Companion 56
Step 9: Do Your Best on Test Day
9. Do Your Best on Test Day
Get ready for test day so you will be calm and condent
You followed your study plan. You prepared for the test. Now its time to prepare for test day.
Plan to end your review a day or two before the actual test date so you avoid cramming. Take a dry run to the
test center so youre sure of the route, traffic conditions, and parking. Most of all, you want to eliminate any
unexpected factors that could distract you from your ultimate goal—passing the Praxis test!
On the day of the test, you should:
be well rested
wear comfortable clothes and dress in layers
eat before you take the test
bring an acceptable and valid photo identification with you
• bring an approved calculator only if one is specifically permitted for the test you are taking (see Calculator
Use, at http://www.ets.org/praxis/test_day/policies/calculators)
be prepared to stand in line to check in or to wait while other test takers check in
You can’t control the testing situation, but you can control yourself. Stay calm. The supervisors are well trained
and make every effort to provide uniform testing conditions, but don’t let it bother you if the test doesn’t start
exactly on time. You will have the allotted amount of time once it does start.
You can think of preparing for this test as training for an athletic event. Once you’ve trained, prepared, and
rested, give it everything you’ve got.
What items am I restricted from bringing into the test center?
You cannot bring into the test center personal items such as:
handbags, knapsacks, or briefcases
water bottles or canned or bottled beverages
study materials, books, or notes
pens, pencils, scrap paper, or calculators, unless specifically permitted for the test you are taking (see
Calculator Use, at http://www.ets.org/praxis/test_day/policies/calculators)
any electronic, photographic, recording, or listening devices
Personal items are not allowed in the testing room and will not be available to you during the test or during
breaks. You may also be asked to empty your pockets. At some centers, you will be assigned a space to store
your belongings, such as handbags and study materials. Some centers do not have secure storage space
available, so please plan accordingly.
Test centers assume no responsibility for your personal items.
The Praxis® Study Companion 57
Step 9: Do Your Best on Test Day
If you have health-related needs requiring you to bring equipment, beverages or snacks into the testing
room or to take extra or extended breaks, you need to request accommodations in advance. Procedures for
requesting accommodations are described in the Bulletin Supplement for Test Takers with Disabilities or
Health-related Needs (PDF).
Note: All cell phones, smart phones (e.g., Android® devices, iPhones®, etc.), and other electronic, photographic,
recording, or listening devices are strictly prohibited from the test center. If you are seen with such a device, you
will be dismissed from the test, your test scores will be canceled, and you will forfeit your test fees. If you are
seen using such a device, the device will be confiscated and inspected. For more information on what you can
bring to the test center, visit www.ets.org/praxis/test_day/bring.
Are You Ready?
Complete this checklist to determine whether you are ready to take your test.
Do you know the testing requirements for the license or certification you are seeking in the state(s) where
you plan to teach?
Have you followed all of the test registration procedures?
Do you know the topics that will be covered in each test you plan to take?
Have you reviewed any textbooks, class notes, and course readings that relate to the topics covered?
Do you know how long the test will take and the number of questions it contains?
Have you considered how you will pace your work?
Are you familiar with the types of questions for your test?
Are you familiar with the recommended test-taking strategies?
Have you practiced by working through the practice questions in this study companion or in a study
guide or practice test?
If constructed-response questions are part of your test, do you understand the scoring criteria for
these questions?
If you are repeating a Praxis test, have you analyzed your previous score report to determine areas where
additional study and test preparation could be useful?
If you answered “yes” to the questions above, your preparation has paid off. Now take the Praxis test, do your
best, pass it—and begin your teaching career!
The Praxis® Study Companion 58
Step 10: Understand Your Scores
10. Understand Your Scores
Understand how tests are scored and how to interpret your test scores
Of course, passing the Praxis test is important to you so you need to understand what your scores mean and
what your state requirements are.
What are the score requirements for my state?
States, institutions, and associations that require the tests set their own passing scores. Visit
www.ets.org/praxis/states for the most up-to-date information.
If I move to another state, will my new state accept my scores?
The Praxis Series tests are part of a national testing program, meaning that they are required in many states for
licensure. The advantage of a national program is that if you move to another state that also requires Praxis tests,
you can transfer your scores. Each state has specific test requirements and passing scores, which you can find at
www.ets.org/praxis/states.
How do I know whether I passed the test?
Your score report will include information on passing scores for the states you identified as recipients of your
test results. If you test in a state with automatic score reporting, you will also receive passing score information
for that state.
A list of states and their passing scores for each test are available online at www.ets.org/praxis/states.
What your Praxis scores mean
You received your score report. Now what does it mean? Its important to interpret your score report correctly
and to know what to do if you have questions about your scores.
Visit http://www.ets.org/s/praxis/pdf/sample_score_report.pdf to see a sample score report.
To access Understanding Your Praxis Scores, a document that provides additional information on how to read
your score report, visit www.ets.org/praxis/scores/understand.
Put your scores in perspective
Your score report indicates:
Your score and whether you passed
The range of possible scores
The raw points available in each content category
The range of the middle 50 percent of scores on the test
If you have taken the same test or other tests in The Praxis Series over the last 10 years, your score report also lists
the highest score you earned on each test taken.
The Praxis® Study Companion 59
Step 10: Understand Your Scores
Content category scores and score interpretation
Questions on the Praxis tests are categorized by content. To help you in future study or in preparing to retake
the test, your score report shows how many raw points you earned in each content category. Compare your
“raw points earned” with the maximum points you could have earned (“raw points available”). The greater the
difference, the greater the opportunity to improve your score by further study.
Score scale changes
E T S updates Praxis tests on a regular basis to ensure they accurately measure the knowledge and skills that are
required for licensure. When tests are updated, the meaning of the score scale may change, so requirements
may vary between the new and previous versions. All scores for previous, discontinued tests are valid and
reportable for 10 years, provided that your state or licensing agency still accepts them.
These resources may also help you interpret your scores:
Understanding Your Praxis Scores (PDF), found at www.ets.org/praxis/scores/understand
The Praxis Series Passing Scores (PDF), found at www.ets.org/praxis/scores/understand
State requirements, found at www.ets.org/praxis/states
The Praxis® Study Companion 60
Appendix: Other Questions You May Have
Appendix: Other Questions You May Have
Here is some supplemental information that can give you a better understanding of the Praxis tests.
What do the Praxis tests measure?
The Praxis tests measure the specific knowledge and skills that beginning teachers need. The tests do not
measure an individual’s disposition toward teaching or potential for success, nor do they measure your actual
teaching ability. The assessments are designed to be comprehensive and inclusive but are limited to what can
be covered in a finite number of questions and question types. Teaching requires many complex skills that are
typically measured in other ways, including classroom observation, video recordings, and portfolios.
Ranging from Agriculture to World Languages, there are more than 80 Praxis tests, which contain selected-
response questions or constructed-response questions, or a combination of both.
Who takes the tests and why?
Some colleges and universities use the Praxis Core Academic Skills for Educators tests (Reading, Writing, and
Mathematics) to evaluate individuals for entry into teacher education programs. The assessments are generally
taken early in your college career. Many states also require Core Academic Skills test scores as part of their
teacher licensing process.
Individuals entering the teaching profession take the Praxis content and pedagogy tests as part of the teacher
licensing and certification process required by many states. In addition, some professional associations and
organizations require the Praxis Subject Assessments (formerly the Praxis II® tests) for professional licensing.
Do all states require these tests?
The Praxis Series tests are currently required for teacher licensure in approximately 40 states and United States
territories. These tests are also used by several professional licensing agencies and by several hundred colleges
and universities. Teacher candidates can test in one state and submit their scores in any other state that requires
Praxis testing for licensure. You can find details at www.ets.org/praxis/states.
What is licensure/certication?
Licensure in any area—medicine, law, architecture, accounting, cosmetology—is an assurance to the public that
the person holding the license possesses sufficient knowledge and skills to perform important occupational
activities safely and effectively. In the case of teacher licensing, a license tells the public that the individual has
met predefined competency standards for beginning teaching practice.
Because a license makes such a serious claim about its holder, licensure tests are usually quite demanding. In
some fields, licensure tests have more than one part and last for more than one day. Candidates for licensure
in all fields plan intensive study as part of their professional preparation. Some join study groups, others study
alone. But preparing to take a licensure test is, in all cases, a professional activity. Because a licensure exam
surveys a broad body of knowledge, preparing for a licensure exam takes planning, discipline, and sustained
effort.
Why does my state require The Praxis Series tests?
Your state chose The Praxis Series tests because they assess the breadth and depth of content—called the
domain”—that your state wants its teachers to possess before they begin to teach. The level of content
knowledge, reflected in the passing score, is based on recommendations of panels of teachers and teacher
The Praxis® Study Companion 61
Appendix: Other Questions You May Have
educators in each subject area. The state licensing agency and, in some states, the state legislature ratify the
passing scores that have been recommended by panels of teachers.
How were the tests developed?
E T S consulted with practicing teachers and teacher educators around the country during every step of
The Praxis Series test development process. First, E T S asked them which knowledge and skills a beginning
teacher needs to be effective. Their responses were then ranked in order of importance and reviewed by
hundreds of teachers.
After the results were analyzed and consensus was reached, guidelines, or specifications, for the selected-
response and constructed-response tests were developed by teachers and teacher educators. Following these
guidelines, teachers and professional test developers created test questions that met content requirements and
E T S Standards for Quality and Fairness.*
When your state adopted the research-based Praxis tests, local panels of teachers and teacher educators
evaluated each question for its relevance to beginning teachers in your state. During this “validity study, the
panel also provided a passing-score recommendation based on how many of the test questions a beginning
teacher in your state would be able to answer correctly. Your states licensing agency determined the final
passing-score requirement.
E T S follows well-established industry procedures and standards designed to ensure that the tests measure what
they are intended to measure. When you pass the Praxis tests your state requires, you are proving that you have
the knowledge and skills you need to begin your teaching career.
How are the tests updated to ensure the content remains current?
Praxis tests are reviewed regularly. During the first phase of review, E T S conducts an analysis of relevant state
and association standards and of the current test content. State licensure titles and the results of relevant
job analyses are also considered. Revised test questions are then produced following the standard test
development methodology. National advisory committees may also be convened to review and revise existing
test specifications and to evaluate test forms for alignment with the specifications.
How long will it take to receive my scores?
Scores for tests that do not include constructed response questions are available on screen immediately after
the test. Scores for tests that contain constructed-response questions or essays aren’t available immediately after
the test because of the scoring process involved. Official score reports are available to you and your designated
score recipients approximately two to three weeks after the test date for tests delivered continuously, or two to
three weeks after the testing window closes for other tests. See the test dates and deadlines calendar at http://
www.ets.org/praxis/register/centers_dates/ for exact score reporting dates.
Can I access my scores on the Web?
All test takers can access their test scores via My Praxis Account free of charge for one year from the posting
date. This online access replaces the mailing of a paper score report.
The process is easy—simply log into My Praxis Account at www.ets.org/praxis and click on your score report. If
you do not already have a Praxis account, you must create one to view your scores.
Note: You must create a Praxis account to access your scores, even if you registered by mail or phone.
* E T S Standards for Quality and Fairness (2014, Princeton, N.J.) are consistent with the Standards for Educational and Psychological
Testing, industry standards issued jointly by the American Educational Research Association, the American Psychological Association, and
the National Council on Measurement in Education (2014, Washington, D.C.).
Your teaching career is worth preparing for, so start today!
Let the Praxis® Study Companion guide you.
Copyright © 2015 by Educational Testing Service. All rights reserved. E T S, the E T S logo, GRE, PRAXIS, PRAXIS II, and THE PRAXIS SERIES
are registered trademarks of Educational Testing Service (E T S). MEASURING THE POWER OF LEARNING is a trademark of ETS.
All other trademarks are property of their respective owners.
To search for the Praxis test prep resources
that meet your specic needs, visit:
To purchase ocial test prep made by the creators
of the Praxis tests, visit the E T S Store:
www.ets.org/praxis/testprep
www.ets.org/praxis/store

Navigation menu